Surgery UWorld

Ace your homework & exams now with Quizwiz!

When to use RETROGRADE CYSTOGRAM?

first of all do RETROGRADE CYSTOGRAM + POST VOID FILMS - is used for DIAGNOSIS OF BLADDER INJURY - may occur FOLLOWING MAJOR TRAUMA ESP W/PELVIC FRACTURE - BUT INDICATION TO DO THIS IS SEEIGN GROSS HEMATURIA

23 y/o m to ED w/painful swollen left knee. Pain began after twisting leg while playing football. Exam shows swollen left knee w/marked tenderness of medial side of knee. When compared to Right knee, valgus stressing the left knee shows exaggerated laxity at joint line. What is next step?

obviously medial collateral ligament- Dx = MRI***

Examination signs in appendicitis

peritoneal signs - rebound tenderness - acute inc in pain after removing hand from applying pressure - involuntary guarding - tensing of abd wall m. during palpation abd. - abdominal rigidity - persistent tension abd wall muscles ***all 3 above = PERITONEAL IRRITATION - significance is RUPTURE OR IMPENDING RUPTURE Psoas Sign - RLQ w/extension R thigh - abscess adj. to psoas or retrocecal appendix Obturator Sign - RLQ pain w/internal rotation of R thigh - pelvic appendix or abscess Rovsing's Sign - RLQ pain w/LLQ palpation + retropulsion colonic contents - acute appendicitis Rectal Tenderness - R pelvic pain during rectal exam - esp w/pressure on r rectal wall - pelvic appendix or abscess

Management of Splenic Trauma

presentation - LUQ pain + tenderness - US show fluid in LUQ - if spleen injury, we see fluid + significant hemorrhage that does KEHR sign where fluid irritates left hemidiaphragm and we see referred left shoulder pain. - DELAYED ONSET HYPOTENSION (one of few abd. organs that can give significant hemorrhage - but presents weirdly) - look for injuries that suggest spleen was involved - ie left lower rib fracture Management - ensure hemodynamic status stable + supply IV fluids. i. if after IV fluids, pt is still unstable - EMERGENT EXPLORATORY LAPAROTOMY - try to save spleen - if removed, vaccinate against encapsulated bacteria ii. if after IV fluids, pt is hemodynamically stable or responds (SBP > 100 mm Hg) - ABDOMINAL CT w/IV contrast

Interventions for lowering ICP

- 3 compartments of brain i. brain parenchyma ii. CSF iii. CBF (cerebral blood flow) these determine ICP - parenchyma + CSF have constant pressures unless space occupying lesion or obstruction to CSF flow. CBF is influenced far more - impt target for tx - mech of hyperventilation - dec cerebral arterial paCO2 - cause rapid vasoconstriction: thereby dec ICP ***impt concept - ventilator rate affect arterial pCO2>>>pO2 versus End Expiratory Pressure in mech ventilated pt or concentration of inhaled O2 affect pO2>>>pCO2

Limb Ischemia (ACUTE ARTERIAL OCCLUSION)

- 80% due to MIGRATORY ARTERIAL THROMBI FROM THE HEART = EMBOLUS - most common sources of atrial thrombi formation = UNDERLYING AFIB OR LEFT VENTRICULAR THROMBUS FROM RECENT MI i. more frequently hit LE >>>> UE - Understand that arterial occlusion can be due to EMBOLUS (MUCH MORE ACUTE ONSET PAIN + ASYMM PULSELESSNESS) versus thrombus is more classic 5Ps in progression - presentation - 5 Ps i. pain ii. pulselessness (EARLY in arterial thrombosis) iii. paresthesia (EARLY) iv. paralysis (LATE) v. pallor (LATE) - sensory changes of subjective nature - paresthesia + tingling - indicate early nerve dysfunction - vs. major motor + sensory deficits - suggest advanced ischemia + nerve damage Tx - 1st step = immediate IV HEPARIN BOLUS - 2nd step - REFERRAL FOR EMERGENCY VVASCULAR SURGERY

Ruptured AAA presentation

- ACUTE ABDOMINA or BACK PAIN followed by SYNCOPE - HYPOVOLEMIC SHOCK ensues*** - widened mediastinum on xray

ARDS due to trauma

- COMPARE WITH PULMONARY CONTUSION - when due to trauma - ARDS OCCURS IN 24-48 HOURS POST TRAUMA VS PULM CONTUSION IN FIRST 24 HOURS ***; ALSO HAS BILAT LUNG INVOVLEMENT (VS PULM CONT HAS UNILAT)****

Diffuse Axonal Injury

- CT show diffuse small bleeds at junctions of grey + white matter - Due to accel/decal shearing forces

27 y/o comes to ED after vehicle accident. No med probs + no meds. BP is 112/92 + Pulse = 122, respiration are 20/min. Mild Resp distress. Bruised on anterior chest wall + abdomen. No murmurs. Breath sounds dec. in left lung base. After initial resuscitation, CXR shows - NG tube up near lungs - what is dx?

- Diaphragmatic Injury Diaphragmatic Rupture

45 y/o F undergoes elective cholecystectomy for gallstones for recurrent severe RUQ pain. After surgery her pain does not resolve and has similar type of pain. During acute pain episode, her labs shows ALT: 80IU/L, AST: 70 IU/L, alk phos of 450IU/L, and a total bilirubin: 1.6 mg/dl. US of abdomen + ERCP are done and show normal bile ducts and pancreas. A sphincter of oddi manometry is done which showed markedly elevated pressure. What would be the most appropriate intervention in this patient?

- ERCP Sphincterotomy

What is next step for any trauma situation based hypotension not responsive to fluid administration?

- Emergent exploratory laparotomy - b/c be concerned about ongoing occult blood loss

Epidural Hematoma vs Subdural Hematoma

- Epidural Hematoma i. accumulation of blood in potential space between cranium + dura mater ii. occurs after moderate degree of trauma to side of head which may result in rupture of MIDDLE MENINGEAL ARTERY as it traverses FORAMEN SPINOSUM iii. Classic: UNCONSCIOUSNESS; FOLLOWED BY LUCID INTERVAL; FOLLOWED BY GRADUAL DETERIORATION OF CONSCIOUSNESS; FOLLOWED BY SX ELEVATED ICP (ex n/v, headache, seizure )- MAY HAVE DILATION OF PUPIL ON SIDE OF LESION DUE TO OCULOMOTOR COMPRESSION + CONTRALATERAL HEMIPARESIS iv. Dx test of choice - CT SCAN - SHOW BICONVEX HEMATOMA - DOES NOT CROSS SUTURE LINES v. Tx - IF THERE ARE NEUROLOGIC SIGNS, EMERGENT CRANIOTOMY TO PREVENT BRAIN HERNIATION Acute Subdural Hematoma (pictured on right) i. head trauma due to TEARING OF BRIDGING VEINS ii. BLEEDING INTO SUBDURAL SPACE iii. may have lucid interval; CT SHOW SEMILUNAR HEMATOMA THAT CROSSES SUTURE LINES

Pulmonary Contusion

- LOOK IN HIGH SPEED CAR ACCIDENT - represent parenchymal bruising of lung - may or may not be associated w/rib fracture - CLINICAL MANIFESTATIONS DEVELOP W/IN 24 HOURS**** - TACHYPNEA, TAHCYCARDIC, HYPOXIA - PE - CHEST WALL BRUISING + DEC BREATH SOUNDS ON SIDE OF PULM CONTUSION - CXR - PATCHY IRREGULAR ALVEOLAR INFILTRATE - UNILAT LUNG INVOLVEMENT**** (CLASSIC DESCRIP***) - ABG - Hypoxemia

What is the simplest way to increase FRC in a post op patient?

- RAISE BED OF HEAD - INC FRC BY 25-30% - PREVENT POSTOP ATELECTASIS

Dude had sex with woman on top - broke his penis - what is next step?

- RETROGRADE URETHROGRAM FOLLOWED BY SURGICAL EXPLORATION OF PENIS

When to use RICE over immobilization?

- RICE for minor ligament tear, not fractures

Fibroadenoma

- Solitary breast lesion - present as painless, firm, and mobile breast mass average around 2 cm in seize - ages 15-25 - benign - DOES NOT change w/MENSTRUAL CYCLE

34 y/o male immigrant from mexico is brought to ED after coughing up foamy sputum w/significant amount of BRB. Clothes stained red. No meds. BP of 112/63 mm Hg, Pulse of 97/min, and RR are 16/min. Pulse Ox shows 95% on room air. Patient is mildly agitated. Breath sounds are audible bilaterally w/some expiratory wheezing on right side. IV fluids are initiated + patient is placed on supplemental oxygen. Portable chest x ray shows dense opacity in upper right lobe - Dx?

- Tb - Next Step: Respiratory Isolation - Reasoning i. pt comes from endemic area (Mexico) ii. upper lobe involvement - should think Tb iii. Radiographic abnormalities of Tb - PATCHY OR NODULAR OPACITY, MULTIPLE NODULES, CAVITY INVOLVING APICAL-POSTERIOR SEGMENTS OF UPPER LOBES OF LUNGS iv. "foamy sputum" + "blood" v. once patient is stable, NEXT STEP = RESPIRATORY ISOLATION UNTIL TB CAN BE CONFIRMED VIA ACID FAST BACILLI SMEAR

How to Manage Variceal Bleeding?

- Think ABCs as usual - 1st step = 2 LARGE BORE IVs OR CENTRAL LINE TO SET UP IV ACCESS FOR FLUIDS - 2nd step (can be first step if airway not already patent) - to prevent risk of aspiration, protect airway via NG DECOMPRESSION VIA NG ASPIRATION - 3rd step - CONTROL BLEEDING ITSELF - in approximately 50% of cases BLEEDING NEEDS NO INTERVENTION; IF NOT THEN MEDICAL OPTION = OCREOTIDE, SOMATOSTATIN, OR TELIPRESSIN; LAST LINE = SCLEROTHERAPY (BAND LIGATION USED ONLY ELECTIVELY)

Next Step in Patient presenting with Blunt Chest Trauma and signs of acute heart failure or shock?

- URGENT ECHOCARDIOGRAM

Features of Compartment Syndrome

- a classic scenario they like to use: patient that has "ACUTE PAIN" + "PARESTHESIA" following a "ISHCEMIA PERFUSION SYDNROME" meaning "POST EMBOLECTOMY" where relative to the underperfused area that it was, the reperfused area now suddenly has a pressure of fluid it is not used to - REPERFUSION OF LIMB FALLOWING ARTERIO OCCLUSIVE ISCHEMIA FOR LONGER THAN 4-6 HOURS GIVES THIS Compartment Syndrome i. due to INTERSTITIAL EDEMA + INTRACELLULAR SWELLING FOLLOWING TISSUE ISCHEMIA - Another scenario where Compartment syndrome is seen - FRACTURE OR INJURY OF EXTREMITIES - ESP LONG BONES - measurement of tissue pressures - diagnostic i. compartment pressure > 30mmHg **** OR ii. delta pressure (diastolic BP - Compartment pressuer) < 20-30 mmHg - Tx - FASCIOTOMY

Acute Mediastinitis

- a common complication of cardiac surgery usually due to INTRAOPERATIVE WOUND CONTAMINATION - look for as complication in STERNOTOMY - typical presentation i. post operatively within 14 DAYS - FEVER, TACHY, CHEST PAIN, LEUKOCYTOSIS, and STERNAL WOUND DRAINAGE OR PURULENT DISCHARGE ii. CXR - shows widened mediastinum (more common in NON POST OP MEDIASTINITIS) iii. may show a little fluid around the heart in echo iv. cloudy fluid at sternal wound drain - treatment of post op mediastinitis - DRAINAGE + SURGICAL DEBRIDEMENT W/IMMEDIATE CLOSURE OF INCISION SITE + PROLONGED ANTIBIOTIC TX

Tension Pneumothorax

- a complication of SUBCLAVIAN CETNRAL VENOUS CATHETER PLACEMENT. - Placement of subclavian central venous catheter accounts for approximately one fourth of iatrogenic pneumothorax. - tension pneumothorax is air within pleural space that dislaces MEDIASTINAL STRUCTURES TO OPP SIDE (TRACHEA) - INC PERCUSSION NOTE (HYPERRESONANT) - rapid onset of SOB, tachycardia, hypotension (COMPRESSION CARDIOPULM), DISTENTION OF NECK VEINS (DUE TO SVC COMPRESSION) AKA JVD - clinical dx - IMMED TX NEEDED - NEEDLE THORACOSTOMY INTO 2nd INTERCOSTAL SPACE TO DECOMPRESS PLEURAL CAVITY FOLLOWED BY EMERGENCY TUBE THORACOSTOMY W/UNDERWATER SEAL - xray not necessary - but would show deviation of trachea = mediastinum away from affected side + inc lucency on affected side of chest. - WHEN CAUSED BY BLUNT THORACIC TRAUMA, MAY BE ASSOC W/BRONCHIAL RUPTURE

Pilonidal Cyst Disease

- acute pain + swelling of midline sacrococcygeal skin + subcutaneous tissues (description of pilonidal cyst) - most prevalent in young males - those with LARGE AMOUNTS BODY HAIR - due to chronic activity involving sweating + friction skin overlying coccyx within superior gluteal cleft - infection of hair follicles in this region then form abscess that ruptures and forms PILONIDAL SINUS TRACT i. chronic sinus tract collects hair follicles + debris giving recurrent infections + foreign body reactions ii. when sinus becomes acutely infected - pain, swelling, and purulent discharge occur in midline postsacral intergluteal region - Tx - DRAINAGE ABSCESS + EXCISION OF SINUS TRACT UWORLD SUMMARY STATEMENT: acute pain + swelling of midline sacrococcygeal skin + subcutaneous tissues - most commonly due to pilonidal dz. Acute presentation involves infection of a dermal sinus tract originating over coccyx.

Paget's Disease of Bone

- aka osteitis deformans - due to INC OSTEOCLAST ACTIVITY - DRASTIC BONE RESORPTION i. causes some degree of inc. osteoblast activity to try and make up - lays down brittle bone (woven bone) - PRONE TO BOWING + FRACTURE - most common presenting sx - pain resulting from bowing or fracture of LONG BONE resulting in SECONDARY ARTHRITIS of HIP OR KNEE - Labs - ISOLATED ELEVATED ALK PHOS - lesions - identified via XRAY OR BONE SCAN - other sx - ENLARGED HAT SIZE (SKULL BONE), HEADACHES, HEARING LOSS (CN PALSIES) - impingement at interal auditory meatus

Aortic Injury

- any patient that suffers from BLUNT DECELARTION TRAUMA (MVA OR FALL FROM >10 ft), MUST R/O BLUNT AORTIC TRAUMA + BLUNT THORACIC TRAUMA - Initial Screening Test - CXR **** - most sensitive finding - WIDENED MEDIASTINUM - may see: displacement trachea +esophagus to the right, downward displacement of l bronchus, even widened aorta

Torus Palatinus (TP)

- benign bony growth = exostosis - on midline suture of hard palate - due to both genetic + environmental factors - more common in younger pts, women, Asians - not associated w/trauma; usually < 2cm + may inc in size thru life - tx = surgery only if symptomatic - b/c may outgrow thin epith + cause irritation, interfere w/speech or eating, problems w/fitting of dentures later in life Differential Dx - Neoplasm of oral cavity - assoc w/smoking or chronic alcohol; tend to grow in size; c an cause sx w/eating or breathing; less likely in younger pt

Presentation of Meniscal versus Ligamentous Tears in Knee

- both ligaments + menisci present with "POP" on rupture - meniscal injuries i. "pop" sensation followed by pain ii. cause joint swelling over following 12-24 hours (VS) iii. PE - JOINT LINE TENDERNESS; (+) MCMURRAY TEST iv. DX = MRI v. occur via activities requiring AXIAL LOADING + ROTATION WHILE IN 30s + 40s - Ligament injuries i. "pop" sensation followed by pain ii. cause joint swelling immediately (VS) due to hemarthrosis - due to better vascular supply than menisci iii. DX = MRI

Peptic Ulcer Perforation

- can also present with similar findings with suddenly onset of severe + diffuse abdominal pain + tenderness. - Metabolic acidosis typically does not occur unless patient develop hemodynamic instability

Anterior Cord Syndrome

- can be due to spinal cord infarction *** i. POSSIBLE COMPLICATION OF THORACIC AORTIC ANEURYSM REPAIR - WHER ANTERIOR SPINAL ARTERY IS DAMAGED a. anterior spinal artery - supply ANTERIOR 2/3s of spinal cord - INCLUDES ANTERIOR CORTICOSPINAL TRACT + SPINOTHALAMIC TRACT (TEMP + PAIN SENS) b. ASA is supplied by artery of adamkiewicz - thoracic aorta surgery can result in damage to this a. then give dec flow thru ASA. ii. spinal shock results - sudden onset BILATERAL FLACCID PARALYSIS + LOSS OF PAIN/TEMPERATURE SENSATION BELOW LEVEL OF SPINAL INJURY iii. UMN signs result - TAKE WEEKS TO DEVELOP - SPASTICITY + HYPERREFLEXIA iv. Sx of Bowel + Bladder dysfunction result - due to INTEROMEDIOLATERAL COLUMN INVOLVEMENT

Acute Vertebral Compression

- can occur in OLDER MEN W/OSTEOPOROSIS - MORE COMMON IN POST MENOPAUSAL WOMEN - Patients classically present with ACUTE BACK PAIN AFTER BENDING, COUGHING, LIFTING, - rare complication - myelopathy - due to retropulsion of bone fragments into spinal canal

Acute Mesenteric Ischemia

- can progress to bowel infarction - presents w/ i. Severe acute onset PERIUMBILICAL pain OUT OF PROPORTION TO PE FINDINGS ii. sudden onset severe poorly localized (DIFFUSE) (visceral) mild abdominal pain accompanied by N/V iii. if bowel infarction occurs - PERITONEAL SIGNS ON ABD EXAM (i.e. tenderness to palpation w/guarding + rebound tenderness); PASSAGE BLOODY STOOL; - most common cause - EMBOLIC OCCLUSION ORIGINATING FROM HEART i. LOOK FOR AFIB IN PT - NO P WAVES + RAPID IRREGULAR PULSE ii. LOOK FOR ATHEROSCLEROTIC RFS + HTN - imaging: CT OR MRA (CT findings listed below); MESENTERIC ANGIOGRAPHY - GOLD STD FOR IMAGING i. focal or segmental bowel wall thickening ii. SMALL BOWEL DILATION iii. MESENTERIC STRANDING RFs i. advanced age ii. AFIB iii. ATHEROSCLEROSIS iv. CHF v. Hypercoaguable disorders Lab FIndings i. Leukocytosis ii. elevated serum Lactate****** (METABOLIC ACIDOSIS) - BICARB LOW + HYPERVENTILATION iii. ELEVATED AMYLASE + PHOSPHATE**** Treatment i. resuscitative measures ii. broad spectrum antibiotics iii. NG tube for decompression iv. Surgery for bowel infarction or perforation MOST COMMON SITE OF OCCLUSION = SMA

DVT

- cause pain + edema of lower extremity - WARMTH TO TOUCH (vs poikilothermia in arterial occlusion) - pain is DULL + ACHING (vs severe + sudden in embolic arterial occlusion)

ALS

- causes upper + lower motor neuron deficits with no loss of sensory function - common sx are muscle weakness + cramping

Umbilical Granuloma

- common cause of UMBILICAL MASS in NEWBORN - appears AFTER UMBILICAL CORD HAS SEPARATED + presents as SOFT, MOIST, PINK, PEDUNCULATED, FRIABLE LESION - Tx of choice = SILVER NITRATE

Stress (hairline) fracture of metatarsals

- common in MILITARY RECRUITS (sudden inc in stress) + athletes - most commonly injured - SECOND METATARSAL - Treatment i. REST, ANALGESIA, HARD SOLED SHOE - Dx - XRAY - 2nd, 3rd, and 4th metatarsals require little treatment besides mentioned b/c surrounding metatarsals act as splints - 5th metatarsal injury - surgical intervention

Foreign Bodies of nose

- common incihjldren = NASAL OBSTRUCTION W/ FOUL ODOR, HALITOSIS, NASAL BLEEDING - following surgery a retained foreign body such as NASAL PACKING - PRECIPITATE CLASSICALLY AS TSS

Penile Fracture

- common to see when women ride man while on top in sexual intercourse - "snapping sensation" - TEARS TUNICA ALBUGINEA **** - HEMATOMA FORMS UPON BREAK - Tx = EMERGENT URETHROGRAM TO ASSESS FOR URETHRAL INJURY + EMERGENT SURGERY TO EVACUATE HEMATOMA + MEND TUNICA ALBUGINEA

Allergic Rhinitis

- commonly present w/rhinorrhea, nasal pruritis, cough, occasionally dyspnea - on exam - nasal mucosa = EDEMATOUS + PALE - polyps may be present

Deep Peroneal Nerve

- deep peroneal nerve - FOOT DORSIFLEXION + TOE EXTENSION; sensation to BETWEEN BIG + 2ND TOE

Postoperative Cholestasis

- develops AFTER MAJOR SURGERY - characterized by HYPOTENSION, EXTENSIVE BLOOD LOSS INTO TISSUES, MASSIVE BLOOD REPLACEMENT - jaundice develops secondary to 3 factors: i. inc pigment load - caused by transfusion ii. dec liver functionality - caused by hypotension iii. dec renal bilirubin excretion - caused by tubular necrosis due to PREERENAL AZOTEMIA FROM HYPOTENSION - Alk Phos may be elevated - ALL OTHER VALUES ARE NORMAL

Injury to Accessory Nerve

- dysfunction due to lesion in medulla -such as occlusion of PICA - Accessory nerve can also be injured during surgical procedure of ANTEROLAT NECK - present with IPS STERNOCLEIDOMASTOID + TRAPEZIUS PARALYSIS

Syringomyelia - aka central cord syndrome

- dz where CSF drainage from the central canal of the spinal cord is disrupted leading to a fluid filled cavity that compresses surrounding neural tissue. - Damage occurs of crossing fibers of spinothalamic tract (pain + temperature) and upper extremity motor fibers due to medial location in corticospinal tract. - most common causes of syringomyelia i. Arnold chiari malformations ii. prior spinal cord injuries (SCI) - if caused by SCI - USUALLY INJURY OCCURS @ CERVICAL SPINAL CORD DUE TO WHIPLASH - Sx develop years after initial injury - classically injury is motor vehicle accident inducing whiplash that manifest years later - characteristic physical exam findings i. dec pain + temperature sensation; loss of motor function (dec strength) affecting arms/hands (CAPE LIKE DISTRIBUTION - where light touch, vibration, and position sense is intact) - MEANING DEC TEMPERATURE + PAIN IN NECK, SHOULDERS, UPPER ARMS, AND HANDS - for this reason may present with "BURNED HANDS" Dx = MRI ****

Postoperative Fever

- either intraoperatively or within 2-6 hours postop, may see malignant hyperthermia (fever, tachycardia, acidosis, and rhabdomyolysis) OR febrile nonhemolytic + acute hemolytic transfusion reactions - COAGULASE NEGATIVE STAPHYLOCCI = most frequent cause of nosocomial bloodstream infection in patients w/intravascular devices i. fever, leukocytosis, hypotension, blood culture growth in > 2 bottles aerobic + anerobic support infection over contamination

Hip Fracture Type/Workup

- fractures are classified by anatomic location + fracture type into either intracapsular (eg femoral neck + head) or extracapsular (ie interrochanteric or subtrochanteric) i. intracapsular fractures - HIGHER RISK OF AVASCULAR NEC ii. extracapsular fractures - greater need for IMPLANT DEVICES IE RODS/NAILS - With Hip fracture - follow trauma protocol - ABCDs but usually real concern is comorbidities. i. in example above, pt somehow unknowingly fell + had dec breath sounds on R side. - DO ECG, CXR, CARDIAC MARKERS TO ENSURE PT STABLE MEDICALLY ii. CAN DELAY SURG UP TO 72 hours if needed to address unstable medical comorbidity.

Meckel Diverticulitis

- generally not direct consequence of blunt abdominal trauma - may mimic appendicitis by causing anorexia + midabdominal/RLQ pain - chills + other signs of sepsis typically do not occur

Routine Use of Abd US?

- highly sensitive for AAA - ONLY DO ROUTINE ONE TIME SCREENING FOR MEN AGE 65-75 w/HX OF SMOKING

Major Causes of morbidity + mortality in significant burns

- hypovolemic shock - in the setting of adequate initial fluid resuscitation, bacterial infection - USUALLY BRONCHOPNEUMONIA OR BURN WOUND INFECTION - LEADING TO SEPSIS + SEPTIC SHOCK = LEADING COMPLICATION

General Anesthesia

- impairs laryngeal defenses - predisposes to aspiration of GASTRIC CONTENTS - residual effects of general anesthesia - can cause hypoventilation - BOTH ASPIRATION + HYPOVENTILATION - usually appear within FIRSTT FEW HOURS OF SURGERY - sx of aspiration = FEVER, COUGH, ACUTE ONSET DYSPNEA< RHONCI/CRACKLES

Acute Cholecystitis Presentation

- inflamm. gallbladder due to obstruction of cystic duct by a gallstone - RUQ PAIN, TENDERENESS, fever, leukocytosis; palpation w/inspiration worse + cause freezing (MURPHY SIGN) - DO EARLY CHOLECYSTECTOMY LAPAROSCOPICALLY WITHIN 72 HOURS OF SX FOR ANY COMPLICS OF GALLSTONES (i.e. gallstone pancreatitis), cholecystectomy. choleldocholithiasis,

Patellar Tendonitis

- jumper's knee - chronic overuse in athletes due to "jumping" - point tenderness over PROX PATELLAR TENDON

Anserine Bursitis

- lies over medial aspect of knee - affect athletes + obese people - attachment of tendons of gracilis, semitendinosus, sartorius

Clavicle Injury Workup

- majority of fractures occur at MIDDLE THIRD of bone - injury typically occurs during athletic events + follows fall on an outstretched arm or direct blow to shoulder - present w/pain + immobility of arm w/contralat hand used to support effed up shoulder - Treat middle third fractures w/REST, ICE, BRACE - All fractures to clavicle - should be accompanied by NEUROVASCULAR EXAM - due to proximity of BRACHIAL PLEXUS + SUBCLAVIAN A - do simple neuro exam for brachial plexus - ROM, etc - for subclavian A, after fracture, if bruit is heard - do ANGIOGRAM - Fracture ot distal third of clavicle i. may require OPEN REDUCTION + INT FIXATION to PREVENT NONUNION

Lymphatic Obstruction Presentation

- may pres at unilateral leg edema - result form malignant obstruction of lymph nodes, lymph node resection, trauma, and/or filiarisis - classically affect DORSA OF FEET + cause MARKED THICKENING + RIGIDITY OF SKIN

Ductal Carcinoma In Situ (DCIS)

- most common in POSTmenopausal women - usually discovered INCIDENTALLY ON MAMMOGRAPY - if symptomatic - NIPPLE DISCHARGE + BREAST MASS - most frequent complaints - Dx - HISTOLOGICALLY - ductal epith abnormalities that do not penetrate basement membrane

scaphoid fracture management

- most commonly fractured bone among carpals - seen in YA due to "FALL ON OUTSTRETCHED HAND" - c/o PAIN AT WRIST JOINT - TENDERNESS AT ANATOMIC SNUFFBOX - SENSITIVE MARKER of scaphoid fracture - susceptible to AVASCULAR NECROSIS - pain on radial aspect of wrist at anatomic snuffbox, minimally dec range of motion (unless dislocated fracture is present), dec grip strength, possible swelling - xray - normal initially or show fine radiolucent lines in nondisplaced scaphoid fractures i. non displaced - fractures w<2 mm of displacement + no angulation - Treatment i. non displaced scaphoid fracture - WRIST IMMOBILIZATION 6-10 WEEKS (TEST Q***) - if xrays are negative, IMMOBILIZE W/SPICA CAST 7-10DAYS + REIMAGE W/XRAY AGAIN (TEST Q***) OR GO DIRECTLY TO CT (more sens)

Injury to glossopharyngeal nerve

- most commonly injured by compression from NEARBY TUMOR as in JUGULAR FORAEMN SYNDROME - posterior fossa tumor. - glossopharyngeal nerve lesion would cause loss of the gag reflex, loss of taste + sensation on posterior one third of tongue, loss of pharyngeal sensation, and dysfunction of carotid sinus reflex leading to an increased risk of syncope

MCL

- most commonly injured ligament of knee - forced abduction knee + torsional action of motion - causes most injuries to this ligament - on exam - the knee joint is swollen due to effusion with tenderness over medial aspect of the knee - MCL resists valgus angulations (abduction) at the knee, injury to this ligament leads to inc. angulation of the affected knee - recall that a ligament injury causes immediate swelling due to hemarthrosis - Dx test of choice = MRI - Tx = bracing + early ambulation

Blunt Abdominal Trauma

- most frequently caused by MOTOR VEHICLE ACCIDENTS - MOST COMMON SOLID ORGANS INJURED - LIVER + SPLEEN **** - presentation - hypotension, tachycardia, chest + abd wall tenderness + ecchymoses after motor vehicle accident - ANYTIME YOU SEE THIS SX AFTER MOTOR VEHICLE ACCIDENT (BAT) OR SEE FREE INTRAPERITONEAL FLUID AFTER BAT, THINK SPLEEN OR LIVER INJURY

Temporal Arteritis Presentation

- new onset headache - temporal tenderness - pulselessness - older age - inc. ESR - may have i. jaw claudication ii. amaurosis fugax

Duodenal Hematoma

- occur following DIRECT BLUNT ABD TRAUMA - more common in CHILDREN - blood collects between SUBMUCOSAL + MUSCULAR LAYERS OF DUODENUM - OBSTRUCTION -presentation - EPIGASTRIC PAIN + VOMITING DUE TO FAILURE TO PASS GASTRIC SECRETION PAST OBSTRUCTING HEMATOMA - most RESOLVE SPONTANEOUSLY IN 1-2 WEEKS - Tx = NG SUCTION + PARENTERAL NUTRITION; if not effective, JTHE NSURGICAL REMOVAL LAPAROSCOPICALLY

Ischemic hepatitis

- occurs for example with LIVER TRANSNPLANT - NOT COMMON B/C LIVER HAS DUAL SUPPLY - signs + sx - nausea, vomiting, and liver enlargement

Cervical Spondylosis

- often in pts > 40 - neck pain + stiffness *** primary sx - develop spinal stenosis - resulting neurologic deficits

Arterial Occlusion Presentation

- pain, pallor. paresthesia, pulselessness, cool to touch (poikilodermia)

46 y/o Man brought to ED after fall during downhill bike race. LOC for 1 minute. C/o severe back + abd pain. No other medical problems. Head CT shows no intracranial bleeding. Lumbar films - show compmression wedge fracture of body of L2 vertebra+ brace is placed. Bd CT - show small retroperitoneal bleed + splenic laceration. On hospital day 3, c/o abd pain + nausea. Abd is distended, tympanic, mildly tender, w/o rebound or guarding. Bowel sounds are absent. AXR - as shows - Dx?

- paralytic ileus - dec bowel sounds, air fluid levels in small + large bowel (vs just small bowel in SBO), trauma causing onset (vs SBO occurs months after surgery)

Complications following Rhinoplasty

- patient dissatisfaction, nasal obstruction, and epistaxis - MAY INVOVLE NASAL SEPTUM - has poor circulation so easy for cartilage loose supply. i. typical presentation is POSTOP - WHISTLING NOISE HEARD DURING RESPIRATION - due to NASAL SEPTAL HEMATOMA OR SEPTAL ABSCESS THAT THEN MAY BECOME PERFORATIONS

SIRS vs Sepsis in Scenario of Severe Burn

- patient w/severe burn - initially manifest some evidence of SIRS + have HYPERmetabolic response in first week after burn i. hyperglycemia - due to insulin resistance ii. muscle wasting iii. protein loss iv. hyperthermia v. inc. energy expenditure - another complication in first week of severe burns - INFECTION (BACTERIA) (TEST Q - being able to recognize signs of this happening****) leading to sepsis + septic shock - main causes are i. pneumonia ii. wound infections due to i. STAPH AUREUS ii. PSEUDOMONAS AEURGINOSA Indications of sepsis are: i. WORSENING HYPERglycemia (more insulin resistance) ii. leukocytosis iii. thrombocytopenia iv. mild hypothermia (temperature < 36C) v. continued tachypnea + tachycardia

Basal Cell Carcinoma

- pearly telangiectatic papules often w/central ulceration

Melanoma

- pigmented + characterized by i. asymmetry ii. border - irregular iii. color - pigmented w/different colors iv. diameter > 6mm v. e - evolution in size + appearance over time etiology - chronic sun exposure - less likely to arise in scars or burns

What to do with amputated finger?

- place amputated finger in saline moistened gauze in plastic bag, put bag on bed of ice, bring it along w/pt to ED. i. DO NOT LET FREEZE THE AMPUTATED FINGER - best candidates are those WITH NO CRUSH INJURY OR AVULSION

PE presentation

- pleuritic chest pain - dyspnea - typically normal xray

Perianal Abscess

- prese w/ANAL PAIN + TENDER ERYTHEMATOUS BULGE AT ANAL VERGE

Breakdown question above

- pt is postop day 4 and w/sx of fever, lung manifestations, think postop pneumonia. - ABG shows acidosis w/low BP + high fever - looks like sepsis. O2 levels are low. Therefore we RECOGNIZE ACIDOTIC ENVIRONMENT DUE TO LACTIC ACIDOSIS IN ANEROBIC RESPIRATION IN AN ATTEMPT TO MAKE UP FOR LACK OF OXYGENATION OF TISSUE AND LACK OF OXIDATVE PHOSPHORYLATION. - once you recognize LACTIC ACIDOSIS W/SEPSIS W/POST OP PNEUMONIA, recognize: i. TREATMENT OF SEPTIC SHOCK - TX UNDERLYING CAUSE + TISSUE PERFUSION RESTORATION = FLUID ADMINISTRATION (0.9% SALINE IV _ ENSURE ISOTONIC, NOT BICARB OR HYPER OR HYPOTONIC) FIRST, THEN GIVE ANTIBIOTICS FOR UNDERLYING SEPSIS ***** - KNOW THIS FOR SEPSIS TX - UWORLD Key Point: "most impt steps in management of lactic acidosis from septic shock are IV normal saline w/or w/o vasopressor tx to maintain the IV pressure + antibiotics to correct underlying infection.

Intraoperative Fluid Loss

- pt w/o preexisting intrinsic kidney dz - oliguria is defined as <400cc or <6cc/kg of urine output per day - if pt after surgery has i. low Hb ii. urine output < 400cc or <6cc/kg per day iii. lower BP iv. oliguria, azotemia, Inc BUN/Cr > 20:1 THINK CAUSE = INTRAOPERATIVE FLUID LOSS When you see OLIGURIA, AZOTEMIA, INC BUN/Cr > 20:1, think 1. urinary catheter obstruction - MUST RULE OUT FIRST then 2. ARF due to prerenal azotemia due to intraoperative fluid loss - NEXT STEP WHEN SEEING PRE RENAL AZOTEMIA (REGARDLESS OF IT BEING DUE TO INTRAOP FLUID LOSSES) - FLUID CHALLENGE VIA IV BOLUS FLUIDS (TEST Q***) - recall pre renal azotemia i. where normal BUN/Cr = 10; here is >20:1 ii. FeNA <1 (distinguishes from intrinsic renal disease which has BUN/Cr > 20:1 but FeNA > 1)

Nasal Polyps seen in?

- pts w/asthma or allergic disorders - patients w/other inflammatory conditions of nasal mucosa see these also - caue CHRONIC NASAL OBSTRUCTION - REMOVE IN SYMPTOMATIC PTS

Radial Nerve Injury

- radial nerve passes through RADIAL GROOVE - POSTERIOR SURFACE OF HUMERUS - most commonly injured nerve in FRACTURE MIDSHAFT HUMERUS OR IMPROPER AXILLARY CRUTCH FITTING - injury can occur during fracture or after reduction of fracture - Radial nerve injury - gives WRIST DROP due to marked limitation of extension at wrist joint + SENSORY LOSS on POSTERIOR ARM, FOREARM, and LATERAL DORSAL HAND

Management of Trauma Based Rib Fractures

- rib fracture - will cause significant PAIN LEADING TO - HYPOVENTILATION - RESULT IN ATELECTASIS OR PNEUMONIA - Therefore priorities = PAIN MANAGEMENT - OPIODS OR NSAIDS = 1st LINE IN MANAGEMENT TO ULTIMATELY PREVENT ATELECTASIS OR PNEUMONIA

Causes of Nasal Septal Perforation

- sarcoidosis, intranasal cocaine use, Tb, syphilis, self inflicted trauma (nose picking), granulomatosis w/polyangitis (Wegener's)

Pulmonary Contusion

- severe blunt chest trauma (in 30-75% there is pulm contusion) can cause direct physical injury to the underlying pulmonary parenchyma + pulmonary contusions. Signs + Sx include dyspnea, tachypnea, chest pain, hypoxemia worsened by intravascular volume expansion (on admin of fluids****), and patchy, irregular alveolar infiltrates on chest x ray i. respiratory alkalosis due to hypoxia driven hyperventilation - management - close monitoring, intubation w/mech ventilation in severe instances

Deep Tissue Infection

- similarly to Compartment syndrome, there is pain out of proportion to obvious clinical signs. - commonly due to GAS OR MIXED ANEROBIC INFXN - Sx evolve over several hours to days + are usually associated w/SKIN NEC, BULLAE, CREPITUS, or signs of Sig SYSTEMIC TOXICITY (HIGH FEVER, HYPOTENSION, ETC)

Superficial Peroneal Nerve

- superficial peroneal nerve - EVERSION OF FOOT; sensation to ANTEROLATERAL LEG + FOOT

Postoperative Atelectasis

- think 2ND POST OP DAY - due ot airway obstruction from retained airway secretions, dec. lung compliance, postop pain, and meds interfering with deep breathing. - ABG - show HYPOXEMIA, HYPOCAPNIA, RESPIRATORY ALKALOSIS - concept is there is hypoxemia due to dec. lung compliance so we breathe more, become hypocapneic + do respiratory alkalosis - presentation - "dec breath sounds at right/left lung base without wheezes or prolonged expiration. Heart sounds are normal. Dense opacity at right/left lung base" - atelectasis - segmental or lobar collapse of lung giving dec. lung volume - may be obstructive or non obstructive Treatment - INCENTIVE SPIROMETRY*** + DEEP BREATHING EXERCISES*** - BEST - epidural analgesia instead of parenteral opiods - continuous positive airway pressure (CPAP) - ADEQUATE PAIN CONTROL W/OPIOIDS - EARLY MOBILIZATION - DIRECTED COUGHING **** KNOW THE ONES IN CAPS - TEST Q*** - MORE COMMON FORLLOWING ABDOMINAL OR THORACOABDOMINAL PROCEDURES

Air under Diaphragm - next step?

- think perforated viscus organ or perforated peptic ulcer - think IMMEDIATE SURGICAL EXPORATION = NEXT STEP

When to use flexible bronchoscopy?

- to evaluate AIRWAY MASSES OR BIOPSY MEDIASTINAL LYMPH NODES

Step of Care in patients with traumatic spinal cord injury

- treat as a trauma patient first - stabilize ABCS i. airway protection + mech ventilation not needed if pt is awake + normal respiratory exam ii. IV access established w/ peripheral IV access (to use 2 large bore IV needles) - do not need additional access i.e. femoral line iii. in ANY TYPE OF NEUROLOGIC DEFICIT ISSUE (LOSS OF SENSATION OR MOTOR FXN), AFTER ABCs, INSPECT FOR BLOOD AT URETHRAL MEATUS (posterior urethral injury) - NOW DO URINARY CATHETERIZATION****** a. prevent urinary retention + acute bladder distention + damage

Flail Chest

- typically due to 3 or more contiguous rib fractures that break in 2 places to create unstable chest wall segment that moves separately from rest of chest - chest wall moves in a paradoxic motion with respiration. ONE SEGEMENT tends to contract during inspiration (normally bulges out) + bulge out during expiration (normally contract) - in PRESENCE OF MAJOR THORACIC TRAUMA + BRUISING TO CHEST - because of associated pain - understand ABG findings - TAKE SHALLOW BREATH, COMPENSATE FOR HYPOXEMIA VIA HYPERVENTILATION - RESPIRATORY ALKALOSIS - management 1. PAIN CONTROL + SUPPLEMENTAL OXYGEN = IMMEDIATLEY 2. NEXT, MECH POSITIVE PRESSURE VENTIALTION w/INTUBATION - should see broken segement now move normally

What factors postoperatively give rise postop atelectasis?

- understand that postop atelectasis is due to

Venous Valve Incompetence

- unilateral lower leg extremity edema that worsen when leg is dependent - improves on leg elevation - venous insufficiency = most common cause of LE edema

Herniated Cervical Disc

- unilateral radiculopathy from compression of nerve root - unilateral pain + weakness in distribution of the involved nerve

Carotid artery screening

- via Doppler - sx of carotid artery occlusion - ocular ischemia/infarction + cerebral hemisphere ischemia (aphasia, visuospatial neglect, apraxia)

Ankle Brachial Index

- when a patient has multiple RFs for athersosclerosis (diabetes, HTN, smoking) and sx consistent w/intermittent claudication - NEXT STEP IN MANAGEMENT - OBSTAIN ANKLE BRACHIAL INDEX (ABI) i. ABI < 0.90 IS DIAGNOSTIC ii. 0.91 - 1.30 - NORMAL iii. >1.30 - CALCIFIED + UNCOMPRESSIBLE VESSELS, ADDITIONAL VASCULAR STUDIES should be considered

Escharotomy Indication

- when circumferential full thickness burns involving extremities or chest are present, ESCHAROTOMY NECESSARY TO PREVENT VASCULAR COMPROMISE + RESPIRATORY DIFFICULTY RESPECTIVELY. - in extremities, may cause compartment syndrome w/6 Ps 1. pain 2. pulselessness 3. paresthesia 4. pallor 5. poikilothermia - another indication: muscle compartment pressure 30mmHg

Ventilator Assisted Pneumonia

- when on assisted ventilation >48 hours - see fever + hypoxia - think PSEUDOMONAS

Transtentorial (uncal) herniation

- when you see patient in trauma situation that presents with CUSHING REFLEX - HTN, BRADYCARDIA, AND RESPIRATORY DEPRESSION - RECOGNIZE THIS IS SIGN OF INC ICP - look for signs - in the case of know EPIDURAL HEMATOMA, IF YOU SEE SIGNS LISTED IN TABLE, CONFIRMS THAT EPIDURAL HEMATOMA CAUSES TRANSTENTORIAL HERNIATION - UWORLD presentation - DUDE WITH RIGH SIDED PARESIS + SUDDEN ONSET OF DROWSINESS WITH HIGH BP - TELLS US ICP ISSUE AND SPEC W/IPS PARALYSIS + DROWSINESS WE THINK UNCAL (TRANSTENTORIAL HERNIATION) - Transtentorial herniation of the parahipoocampal uncus during significant head injury leads to i. ipsilateral hemiparesis ii. ipsilateral mydriasis iii. ipsilateral strabismus (outside + out) iv. contralateral hemianopsia v. altered mentation

Fibrocystic Changes

-very common in premenopausal females. - pts present w/BILAT breast pain assoc w/cystic changes of breasts - condition = BENIGN - sx vary cyclically w/menstrual cycle - PE- lumpiness of breasts

Rupture of Tendon of Long Head Biceps Pres?

1. "POPEYE SIGN" - biceps muscle belly becomes prominent in mid upper arm

Nasopharyngeal Carcinoma

1. - undifferentiated carcinoma of SQUAMOS CELL ORIGIN 2. SEEN IN HIGHER FREQUENCY IN MEDITERRANEA OR FAR EASTERN DESCENT 3. MOST NPC = METASTATIC AT TIME OF DX 4. presentation - RECURRENT OTITIS MEDIA (Eustachian tube blockage by tumor), RECURRENT EPISTAXIS, NASAL OBSTRUCTION 5. associated with i. EBV VIRUS **** ii. SMOKING iii. NITROSAMINE INTAKE (SALTED FISH)

Parotid Neoplasms

1. 2 lobes of parotid gland present. Separated by FACIAL NERVE (VII) 2. Removal of deeper parotid gland lobe - require dissection of branches Facial Nerve VII - UNILATERAL FACIAL DROOP 3. "non tender swelling in cheek" w/exam of "PREAURICULAR SPACE FULLNESS"

Humeral Neck fracture - can involve what nerve? Pres?

1. Axillary nerve 2. Pres w/swelling, ecchymosis, crepitus over fracture

Most common site of extraperitoneal bladder rupture?

1. Bladder Neck

Primary Hypoparathyroidism

1. Hypoparathyroidism - parathyroid hormone deficiency is characterized by hypocalcemia + hyperphosphatemia in presence of normal renal function 2. CFs i. neuromuscular irritability - perioral tingling + numbness, muscle cramps, tetany, carpopedal spasms, seizures ii. prolongation of QT 3. Causes i. post surgical - most common cause - occur during thyroidectomy or sub total parathryoidectomy ii. autoimmune - MOST COMMON NON SURGICAL CAUSE iii. congenital absence or maldevelopment (digeorge syndrome) iv. defective Ca2+ sensing Receptor on PTH glands v. Non-autoimmune destruction of the Parathryoid gland due to infiltrative dz like Wilson's dz, hemochromatosis, neck irradiation

55 y/o M comes to ED w/gunshot to abd. Had laparotomy. On postop day 4, waxing + waning fever, tachypnea, SOB. Temp is 104, BP is 90/60, pulse is 110, and respirations are 22/min. PE - crackles in R lung base + well healing abd wound. UO over last 12 hours is 100mL. Lab profile: Blood pH - 7.23; PaO2 - 60; PaCO2 - 32 mmHg; HCO3- - 16mEq/L - antibiotics are started + what other tx is necessary?

1. IV 0.9 % SALINE****

Hemoptysis Management

1. Initial management i. FIRST ESTABLISH PATENT AIRWAY ii. SECOND ENSURE HEMODYNAMIC STABILITY + PLACE PATIENT ON BLEEDING LUNG SIDE (DEPENDENT POSITION) iii. IF BLOOD > 600mL/day or 100mL/hour, NEXT STEP = BRONCHOSCOPY TO localize bleeding site, provide suctioning ability, therapeutic options (ie. balloon tamponade + electrocautery OR pulmonary embolization)

Hoarseness is a side effect of ?

1. Injury to RECURRENT LARYNGEAL BRANCH OF VAGUS 2. VULNERABLE TO DMG in SURGERY OF PARATHRYOID OR THYROID

Acalculous Cholecystitis

1. LOOK FOR SEVERELY ILL PATIENT - where it most commonly occurs i. pts w/surgery, severe trauma, burns, sepsis, prolonged parenteral nutrition 2. due to CHOLESTASIS + GALLBLADDER ISCHEMIA GIVING SECONDARY INFECTION + RESULTING IN EDEMA + NECROSIS GALLBLADDER i. can lead to SEPSIS 3. Clinical signs - vague - fever + leukocytosis 4. Radiologic Signs - GALLBADDER WALL THICKENING + DISTENTION + PERICHOLECYSTIC FLUID ******* 5. Tx = ANTIBIOTICS + PERCUTANEOUS CHOLECYSTOSTOMY UNDER RADIOLOGIC GUIDANCE i. CHOLECYSTECTOMY when STABILIZED 6. Dx - US

Aneurysms

1. Most common - AAA, POPLITEAL ARTERY ANEURYSM, FEMORAL ARTERY ANEURYSM 2. Peripheral artery aneurysm manifest as pulsatile mass that can compress adjacent structure (nerves - cause pain, vein). Can result in thrombosis and ischemia. Popliteal + femoral a. aneurysm are most common peripheral artery aneurysms. i. FREQ ASSOC W/ABDOMINAL AORTIC ANEURYSMS

If Pt present with thigh furuncle infection of hair follicles and then has some deep tenderness to palpation of RLQ on abd exam, what do you think?

1. Musculoskeletal infections such as osteomyelitis or abscess frequently result from hematogenous spread of organisms from another site such as skin (FURUNCLES IN THIS CASE) 2. S AUREUS is most common offending organism. 3. CT IS DX TEST OF CHOICE OF INTRAABDOMINAL ABSCESS 4. Probably not appendicitis (no typical sx) - so NEXT THOUGHT = PSOAS ABSCESS i. Tx = drainage with PERCUTANEOUS DRAINAGE TUBE

1. `1 y/o m ocmes to ED after direct blunt trauma to upper abdomen. Has epigastric pain + repeated vomiting immediately after trauma. Is afebrile + other vital signs are stable. Barium exam - show duodenal obstruction. CT abdomen - duodenal hematoma. Next step?

1. NG suction + parenteral nutrition

Pancreatitis Findings on CT

1. PARENCHYMAL ENHANCEMENT W/IV CONTRAST, PSEUDOCYST FORMATION, OR PERIPANCREATIC FLUID COLLECTION

Gastric Outlet Obstruction

1. Present with N + V, EARLY SATIETY, EPIGASTRIC PAIN OR DISTENTION 2. patients with prolonged or recurrent vomiting - HYPOKALEMIA OR HYPOCHLOREMIC METABOLIC ALKALOSIS 3. PE i. abdominal sucussion splash - elicited by placing stethoscope over the upper abdomen + rocking patient back + forth at hips. Retained gastric material >3 hours after meal - generate splash sounds + indicate presence of hollow viscus filled with both fluid + gas 4. Common causes i. gastric malignancy ii. peptic ulcer disease iii. crohn's dz iv. stricture (w/pyloric stenosis) secondary to ingestion of caustic agents - i.e. acid ingestion - causes fibrosis 6-12 weeks after the resolution of acute injury - Upper endoscopy used to confirm dx - tx = surgical

Differential for Hemoptysis

1. Pulmonary I. bronchitis ii. PE iii. bronchiectasis iv. Lung Cancer 2. Cardiac i. mitral stenosis/acute pulmonary edema 3. Infectious i. Tb ii. Lung Abscess 4. Hematologic i. coagulopathy 5. Vascular - arteriovenous malformations 6. Systemic Dz i. Wegener granulomatosis ii. Goodpasture syndrome iii. SLE, Vasculitis

Postoperative Pneumonia + prevention

1. RFs are numerous for postop pneumonia i. thoracic or abdominal surgery ii. age > 50*** iii. long invasive surgery > 3hours iv. SMOKING v. poor general health Prevention post op pneumonia i. incentive spirometry - 1st LINE PREVENTATIVE MEASURE USED POSTOPERATIVELY ii. deep breathing exercises *** ABOVE 2 ARE BEST iii. continuous Positive airway pressure (CPAP) iv. intermittent positive pressure breathing

Duodenal Perforation

1. Sudden Onset, DIFFUSE ABD PAIN 2. ABdomen - rigid on initial exam; distended later w/signs of peritonitis 3. CXR - FREE AIR UNDER DIAPHRAGM (LIKE DUODENAL ULCER)

TEE is test of choice for?

1. Suspected Aortic Injury 2. Hemopericardium 3. Cardiac Tamponade

Differential Diagnosis of Anterior Mediastinal Mass

1. Thymoma 2. Teratoma i. include other germ cell tumors ii. if teratoma, look for CALCIUM OR FAT (PERHAPS IN FORM OF A TOOTH) iii. Seminomatous tumors - B-HCG MAY BE INC; AFP NORMAL (BS - for b-hcg + seminoma) iv. Non-seminomatous tumor - yolk sac tumor, choriocarcinoma, embryonal carcinoma - INC AFP; MAY HAVE INC B-HCG ALSO 3. Thyroid Neoplasm 4. Terrible lymphoma

When To Insert Chest Tube?

1. When you see findings of hemothorax

Diaphragmatic Rupture Presentation

1. abd. pain referred t shoulder 2. SOB + Vomiting 3. CXR - abd viscera above diaphragm + loss of diaphragmatic contour

Most common site of intraperitoneal bladder rupture?

1. bladder dome - present w/chemical peritonitis causing shoulder pain due to urine in peritoneal cavity impinging of parietal hemidiaphragm

NG tube placement indication

1. bowel obstruction 2. enteral nutrition 3. gastric lavage - to assess current bleeding episode 4. to assess upper versus lower GI bleed

Erosive Gastritis

1. can be complication of trauma 2. present with ABD Discomfort, N, V, hematemesis

Long Thoracic Nerve Injury - How?

1. cause WINGED SCAPULA 2. paralysis of SERRATUS ANTERIOR 3. Most common cause = IATROGENIC INJURY DURING AXILLARY LYMPHADENECTOMY

Acute Appendicitis

1. clinical dx 2. classic presentation - migratory pain from periumbilical (due to viscera) to localizing to RLQ pain, N/V, fever, leukocytosis McBurney Point tenderness (1/3 way up from ASIS), Rovsing sign (press LLQ, feel pain at RLQ) 3. Once Clinical Dx made - NEXT STEP = LAP APPENDECTOMY (ALWAYS ANSWER ON TEST) i. IMAGING LIKE CT OR US IN NEXT STEP ONLY IF SX NOT CLASSIC

Brachial artery injury

1. common in SUPRACONDYLAR FRACTURE OF HUMERUS (along with possible median nerve injury) 2. sx - pain, pallor, pulselessness, paresthesia, pressure i. LOSS OF BRACHIAL A + RADIAL A PULSES

Indication for NG tube placement?

1. differentiate between Upper + lower gi bleed. 2. to detect ongoing bleeding - red blood in lavage 3. to remove particulate matter

Postpericardiotomy Syndrome

1. fever, leukocytosis, tachycardia, and chest pain 2. usually AutoImmune - occurs few weeks following procedure w/pericardium incision 3. Tx = NSAIDs, steroids treat inflammation; pericardial puncture if tamponade occurs

Nasal Furunculosis

1. from staph folliculitis - following NOSE PICKING OR NASAL HAIR PLUCKING - POTENTIALLY LIFE THREATENING as it can spread to CAVERNOUS SINUS - c/o PAIN, TENDERNESS, ERYTEHMA NASAL VESTIBULE

Tracheobronchial Injury

1. hemoptysis 2. subcutaneous emphysema (palpable crepitus below skin) 3. air leak (PNEUMOTHORAX) + PNEUMEDIASTINUM even after chest tube placement*** 4. Right Main bronchus - more likely injured

Chronic Pericardial Tamponade

1. in contrast, chronic processes like MALIGNANCY OR RENAL FAILURE can cause slow accumulation of pericardial fluid that gradually inc. intrapericardial pressure + allow pericardial elasticity to adapt slowly 2. As a result, may take 1-2 L of fluid (vs few mL in acute setting) b/f classic picture as above occurs 3. See changes in CXR in this situation - classic GLOBULAR HEART SHAPE

What is major complication of infection of PARAPHARYNGEAL SPACE?

1. involvement of carotid sheath - give erosion of carotid artery giving JUGULAR THROMBOPHLEBITIS

Median Nerve Injury

1. loss of sensation of skin over lateral 3.5 fingers + lateral half of palm

Gallstone Pancreatitis

1. midepigatric pain, N/V, pain w/radiation to back i. all tell us this is pancreatitis 2. Labs - INC AMYLASE + LIPASE i. to see if it is gallstone induced, CHECK ALT - IF ALT>150, then this is gallstone induced 3. Tx - when due to gallstones, MUST DO CHOLECYSTECTOMY WITHIN 72 HOURS

Appendicitis

1. migratory - vague periumbilical to sharp RLQ pain - abd pain, fever, nausea, vomiting, and anorexia. 2. pts may have signs of irritation of parietal peritoneum - rebound tenderness, involuntary guarding, abdominal rigidity - due to IMPENDING RUPTURE OR COMPLETE RUPTURE APPENDIX 3. Tx IF PERITONEAL SIGNS (indicating impending or already happened rupture) = URGERY APPENDECTOMY 4. pts w/delayed presentation - w/longer duration of sx (>5 days) often have appendiceal rupture with CONTAINED abscess. i. SIG FEVER, LEUKOCYTOSIS ii. Tx - IV ANTIBIOTICS, BOWEL REST, PERCUTANEOUS DRAINAGE OF ABSCESS; RETURN 6-8 WEEKS AFTER FOR APPENDECTOMY ELECTIVELY - "INTERVAL APPENDECTOMY" 5. Dx - clinical, confirm with CT***

Rotator Cuff Tears

1. most commonly injured = SUPRASPINATUS i. due to repeated BOUTS OF ISCHEMIA NEAR INSERTION ON HUMERUS INDUCED BY COMPRESSION BETW HUMERUS + ACROMION ii. common prez = FALL ON OUTSTRETCHED ARM 2. Prez = sharp shoulder pain + edema following trauma + UNABLE TO ABDUCT ARM PAST 90 degrees 3. (+) DROP ARM TEST - pt's arm abducted passively to 90 degrees + when pt asked to lower arm slowly, IF COMPLETE CUFF TEAR, PT UNABLE TO LOWER ARM SMOOTHLY AND WILL DROP RAPIDLY FROM NEAR 90 DEG POSITION

Diabetic Ketoacidosis

1. nausea, vomiting, abdominal pain, and tenderness with ANION GAP METABOLIC ACIDOSIS 2. DKA usually occurs in patients with TIDM w/plasma glucose levels of 250-800 mg/dL. 3. NO peritoneal signs

Myocardial Contusion presentation

1. new bundle branch block or arrhythmia 2. see tachycardia 3. STERNAL FRACTURE - ASSOCIATION 4. PCWP INC A LOT - HIGH PRELOAD BUT PUMP NOT FUNCTIONING (CAN PRECIPITATE CARDIOGENIC SHOCK )

What is treatment first line for a patient who has R calf pain since 3 days ago and had a surgery recently a week ago. Has pulse of 92, Normal temp, BP of 120/70. Ankle edema in right + Duplex showing clot in distal portion of femoral vein.

1. note supercficial thrombosis DOES NOT NEED ANTICOAGULATION 2. In the case of a provoked DVT as in this patient (post surgery - surgery was cause) 3. treatment for DVT that had clear reasoning for its cause is i. HEPARIN - bridge for 4-5 days ii. THEN WARFARIN FOR >3 MONTHS WITH INR of 2-3 4. Guidelines for Antiocoagulation of DVT post surgery i. good to start 48-72 hours post surgery without any significantly increased risk of bleeding or any complication

Compartment Syndrome

1. occur following a fracture due to inc. in compartment pressure secondary to edema and/or bleeding 2. 5 P's - pain, pallor, pulselessness, paresthesia, and pressure 3. most common in CALF OR FOREARM Most Common Presentation - EXCRUCIATING PAIN worsened by passive stretch - pain inc. on passive stretch - rapidly increasing + tense swelling - paresthesias (EARLY) Uncommon Presentation - dec sensation + motor weakness (within hours) - paralysis + dec distal pulses (late) Etiology 1. prolonged compression of extremity 2. after revascularization of acutely ischemic limb Most Important Prognostic Factors - TIME TO FASCIOTOMY - DO NOT DELAY ON DX THE SURG TX Next Step on suspicion - COMPARTMENT PRESSURE MUST BE MEASURED - if this is not a choice, THEN CHOOSE IMMEDIATE FASCIOTOMY -

Injury to Facial Nerve at brain level

1. occurs at level right between PONS + MEDULLA - cause UMN lesion - contralateral lower facial droop****

Aortic Dissection

1. patients suffering from RAPID DECELERATION BLUNT CHEST TRAUMA ARE AT HIGH RISK FOR AORTIC INJURY 2. "SUDDEN TEARING + excruciating chest pain started hours ago"; "RADIATES TO BACK OR NECK'' 3. CXR i. WIDENED MEDIASTINUM ii. large LEFT SIDED HEMOTHORAX iii. DEVIATION MEDIASTINUM TO RIGHT + DISRUPTION AORTIC CONTOUR *** TO CONFIRM DX DO CT w/contrast or TEE 4. May see >20 mm Hg variation in systolic BP between arms 5. IF AORTIC ROOT INVOLVED - LOOK FOR AORTIC REGURG MURMUR - DIASTOLIC DECRESCENDO 6. Variety of Risk Factors for development of Aortic Dissection i. HTN - MOST COMMON ii. Marfan + Ehler Danlos - present w/dissection in YOUNGER PT iii. Cocaine use 7. Complications (involved structure) i. CAROTID ARTERIES - stroke ii. Aortic valves - Aortic Regurg iii. Superior cervical sympathetic chain ganglion - Horner's syndrome iv. If coronary artery - Acute MI v. Pericardial Cavity - pericardial effusion vi. Pleural Cavity - hemothorax (LEFT SIDED w/MEDIASTINAL SHIFT TO THE RIGHT) vii. spinal or common iliac arteries - LE weakness or ischemia viii. mesenteric artery - abd. pain 8. CXR findings - mediastinal widening - depression l main bronchus - displacement trachea + esophagus to the right - obliteration of the aortic knob shadow 6. CONFIRM WITH CT

Esophageal Rupture Presentation

1. pneumomediastinum + pleural effusion 2. dx confirmed with WATER SOL ESOPHAGOGRAM 3. Circulatory Collapse is not seen 4. subcutaneous emphysema 5. potentially shock via MEDIASTINITIS + THIRD SPACING INTRAVASCULAR FLUID INTO MEDIASTINUM 6. RETROSTERNAL CHEST PAIN 7. cause i. iatrogenic - endoscopy ii. esophagitis related

Fat embolism presentation

1. polytrauma - especially fractures of LONG BONES 2. present with SEVERE RESPIRATORY DISTRESS, PETECHIAL RASH, SUBCONJUNCTIVAL HEMORRHAGE, TACHYCARDIA, TAHCYPNEA, FEVER, CNS DYSFUNCTION 3. Dx confirmation i. FAT DROPLETS IN URINE OR ii. intra arterial fat globules on FUNDOSCOPY 4. occur 12-72 hours post injury 5. CNS dysfunction initially manifests as confusion + agitation - may manifest 6. XRAY - BILATERAL PULMONARY INFILTRATE WIHTIN 24-48 HOURS OF ONSET OF CLINICAL FINDINGS 7. Thrombocytopenia, hypofibroginemia, anemia ***

GCS - Glasgow Coma Scale

1. predict severity + prognosis of coma during primary survey 2. 3 components i. ABILITY TO OPEN EYES ii. MOTOR RESPONSE iii. VERBAL RESPONSE

Recurrent Embolism

1. present possibly like Compartment syndrome - with 5 Ps 2. differentiate from CS via LACK OF LOCAL SWELLING

Mets to Bone

1. present with pain + tenderness. 2. complicated often by pathologic fractures 3. often more common than primary bone tumors 4. affect PROXIMAL HUMERUS OR FEMUR 5. Radiographic evidence of bone destruction or tumor - usually evident on x ray in these cases

Acute Bacterial Parotitis

1. present with: i. painful swelling of parotid gland AGGRAVATED BY CHEWING ii. HIGH FEVER iii. tender, swollen, erythematous parotid gland - PURULENT SALIVA iv. OFTEN A POSTOP COMPLICATION - PREVENT VIA FLUID HYDRATION + ORAL HYGIENE ******* - due to STAPH AUREUS ****** 2. Common Population i. OLD + POST OP PTS THAT ARE DEHYDRATED *** - present as old post op pt w/parotid gland swelling - dx is a gimme

Femoral Artery Aneurysm

1. pulsatile groin mass below inguinal ligament 2. anterior thigh pain may be present unilaterally due to compression femoral nerve running laterally to artery.

What other conditions cause precipitation of Ca2+ as soaps or in tissue w/PO43-?

1. renal failure, rhabdomyolysis, phosphate administration all have inc PO43- and cause precipitation as salts 2. In Acute pancreatitis OR diffuse osteoblastic mets (prostate + breast cancer), precipitation of calcium soaps in abd cavity

Bone Infarction

1. seen in patient w/sickle cell in vaso occlusive episodes

AAA

1. severe back pain, syncope, hypotension - think AAA 2. >3 cm = aneurysm (normal diameter is <3cm) 3. AAA - involves all layers - not a flap or false lumen 4. occurs in people > 60 years of age, smokers, men, people w/hx of CAD 5. PE - may show pulsating abdominal mass @ or above umbilicus 6. rupture cause death 50% of time + present w/SEVERE + SUDDEN HYPOTENSION i. CAN HAVE ECG CHANGES IN RUPTURE - ST DEPRESSIONS - indicating ischemia 7. possible complication OF RUPTURE includes AORTOCAVAL FISTULA W/IVC GIVING VENOUS CONESTION IN RETROPERITONEAL STRUCTURES IE BLADDER - MAY CAUSE DISTENDED VEINS IN BLADDER TO RUPTURE - GROSS HEMATURIA 8. Diff dx = Mass Pe - but LACK OF DISTNDED NECK VEINS + TACHY w/already existing Hypotension tells us this is not it

Tic Doloureux (trigeminal neuralgia)

1. short burst of excruciating, lancinating pain lasting from SECONDS TO MINUTESE in distribution of 2nd + 3rd branches of TRIGEMINAL NERVE 2. DUE TO EXTERN COMPRESSION

What is post cholecystectomy pain due to?

1. sphincter of oddi dysfunction - charac. by elevated biliary sphincter pressure; RUQ pain + elevation of liver enzymes during pain attack (dec. to norm when pain resolves) i. Tx = ERCP with Sphincterotomy 2. Common bile duct stones 3. Or functional pain - first step in post cholecystectomy pain - DO US + ERCP - to r/o common bile duct stones

65 y/o M with Total Knee replacement 6 months ago comes in complaining of 3 weeks of dull pain in his right knee. T = 99.2, BP of 120/60, and pulse is 90/min. Exam shows mild swelling + tenderness over right knee. There is full ROM. Skin over joint is intact. WBC count = 10,000/uL w/80% neutrophils. Which of the following is the most likely cause of current condition?

1. staphylococcus epidermidis

Differential Diagnosis for unilateral hip pain in middle aged adults

1. trauma 2. infection 3. bursitis 4. radiculopathy

When to use ERCP?

1. when there is CBD INVOLVEMENT - dilation, obstruction, or inc. liver enzyme levels. 2. allows cannulation, sphincterotomy in an attempt to relieve obstruction

When Is Bronchoscopy an option?

1. when there is massive hemoptysis either >600mL/day or >100mL/hr (i.e. in Tb example if pt was coughing up blood >600mL/day then we know to do bronchoscopy)

Management of Cervical Spine Trauma

1st step i. stabilize cervical spine + spinal column w/backboard, rigid cervical collar, or lateral head supports 2nd step i. follow on chart - four people required for orotracheal intubation with rapid sequence intubation i. first person manually stabilizes patient ii. 2nd person administers induction anesthesia iii. 3rd patient applies cricoid pressure to prevent passive regurgitation until tube placement confirmed iv. 4th places endotracheal tube When is laryngeal mask used? - to stabilize patient until another airway can be established IF OROTRACHEAL FAILS Nasotracheal Intubation contraindicated in what patients? - APNEIC OR HYPONEIC PTS - if PT HAS BASILAR SKULL FRACTURE - cribiform plate disruption - gives inadvertent passage tube into brain

Halothane Hepatotoxicity

2 Types 1. Type I - associated with aminotransferase elevation w/mild or no sx 2. Type II - associated w/SEVERE FULMINANT HEPATITIS -

31 y/o biker involved in motor vehicle accident. Describes direct blow to lower abdomen + pelvis during accident. Complains of diffuse abd. pain referring to left shoulder. Which of the following injury most likely accounts for current sx?

A: Bladder Dome Injury Reasoning - abd. pain referring to shoulder = suggest irritation of diaphragm via peritonitis (KEHR SIGN) - in setting of blunt abd. trauma, hemoperitoneum, spillage of bowel contents, bile, pancreatic secretions, or urine - can cause acute chemical peritonitis. - of choice given of which were urethra related, bladder related, or kidney related, only logical answer was DOME OF BLADDER b/c i. dome of bladder - most likely to rupture given sudden increase in intravesicular pressure ii. dome of bladder - ONLY PART OF BLADDER COVERED BY PERITONEUM AND WOULD THEREFORE RELEASE URINE INTO PERITONEAL CAVITY

25 y/o brought to ED after road traffic accident occurring 90 mins ago. Was in front seat of passenger seat. Initial BP at accident was 90/60 + Pulse of 126. In ED BP of 110/70, pulse of 90/min. Abdomen is tender in LUQ. US - fluid in spleno-renal angle. Most appropriate next step is?

A: CT scan Reasoning - fluid in LUQ + Pain - think of spleen - spleen is most fragile organ that gives rise to clinically significant bleeding

73 y/o m comes to ED. Fell + L hip hurts a lot. Can't remember falling. Over past several days, has had SOB w/cough + intermittent palpitations attributed to "flu." PMH is HTN, TIIDM< COPD< BPH. VSSAF. Pulse Ox shows 95% on 2L/min of O2 by nasal cannula. LLE shortened + ext rotated. What is next best step in pt?

A: ECG, cardiac markers, CXR

63 y/o present to urgent care center w/4 hour history of abdominal pain described as severe, diffuse, constant. Has had 1 episode non bloody vomiting since pain started. PMH for hyperlipidemia, TIDM, HTN, Afib, kidney dz. Meds are Lisinopril, digoxin, warfarin, Metoprolol, simvastatin, insulin glargine. BP is 130/70 mmHg and HR is 100/min + irreg. PE reveals overweight male in moderate distress. Abdomen diffusely tender to palpation w/ (+) rebound tenderness. Labs are as follows. Platelets of 90,000; Hb of 9.5; WBC of 7500. Glucose of 210. INR of 2.1. Blood digoxin level that is therapeutic. Abd xray show air under diaphragm. Which of the following is the best initial treatment for this patient?

A: Fresh Frozen Plasma - in a patient with acute GI perforation - NEED EMERGENT LAPOROTOMY - BUT PRIOR TO THIS MUST STABILIZE + CORRECT INR IF ON WARFARIN. - Step in prepping for emergent laparotomy i. pre operative NG tube decompression, IV fluids, antibiotics ii. next step = WARFARIN INDUCED ANTICOAG MUST BE REVERSED - to do so acutely ADMINISTER FFP**** (TEST Q***) iii. correct simultaneously blood or platelet issues: IF HB < 7 THEN INFUSE W/PRBCs + IF PLATELETS<50,000 INFUSE W/PLATELETS

64 y/o man brought to ED after motor vehicle accident - he was restrained driver when his car crashed into a moving car in front of him. Resuscitated w/2 L of normal saline on way to hospital. he has not lost consciousness. Med probs include CAD + epilepsy. Underwent coronary stent placement 2 years ago. Meds include antiepileptic agents. His BP is 82/52 + pulse is 123 + regular. Pulse ox shows 96% at room air. He is lethargic + slowly but appropriately answers in answering. Trachea midline, Bilateral breath sounds present. Abdomen is mildly tender. What is next step - head CT w/o contrast, chest ct w/contrast, intraperitoneal free fluid detection, LV EF assessment, Pancreatic enzymes measurement?

A: Intraperitoneal free fluid measurement - For such hypotension - has to be either pelvis break or abdomen that is giving way to such hemorrhage - not chest because it would be symptomatic. If at pelvis or abdomen, minimal sx. ***** i. abd was tender - so look for intraperitoneal free fluid - first step after fluid resuscitation - determine if a patient needs exploratory laparotomy VIA FAST ****** (KNOW THIS) i. when combined US of pericardium + abdomen, called FAST (focused assessment with sonography for trauma) examination. ii. can detect hemoperitoneum, pericardial effusion, and intraperitoneal fluid. iii. if FAST exam equivocal, DO DPL FOR HEMOPERITONEUM - iv. IF FAST OR DPL ARE POS THEN DO EXPLORATORY LAP Summary: Pts w/blunt abd trauma w/some indication that have hypotension should undergo FAST, then DPL if equivocal, THEN IF (+) EXPLORATORY LAP

28 y/o M brought to ED 4 hours after being involved in motor vehicle collision. BP of 90/50 mm Hg, pulse of 120, RR of 30/min. Exam - stuporous man w/bruises over extremities + upper abdomen. Trachea = midline + neck veins are flat. Abdomen distended + NTTP. After intubation + mech ventilation, goes into cardiac arrest. Which of the following would have prevented cardiac arrest in this patient?

A: Volume resuscitation - FLAT NECK VEINS = HYPOVOLEMIA - in combination, flat neck veins + significant bruising + abd distention on PE = HYPOVOLEMIC SHOCK DUE TO HEMORRHAGE MOST LIKELY DX - remember - severe hemorrhage - CAUSE DEC VENOUS RETURN + EDV + CO. AS A RESULT, VENOCONSTRICTION OCCURS. - Positive Pressure Mechanical Ventilation - INC INTRATHORACIC PRESSURE - INC RIGHT ATRIAL PRESSURE - DEC SVR. - DEC PRELOAD + EDV - MAY CAUSE ACUTE CIRCULATORY FAILURE + DEATH - Sedatives - RELAX VENOUS RETURN - DEC EDV + CO - PEEP (positive end expiratory pressure) - same effect as positive pressure mech ventilation but to lesser degree

84 y/o come to ED w/1 hour back pain, syncope lasting < 1 minute, and an episode of gross hematuria prior to coming to hospital. has SOB, no chest pain, cough, N/V, pulse ox at 92% room air. ECG show prominent horizontal ST segment depression in anterior chest leads.. What is probably diagnosis?

AAA rupture

If you see ABDOMINAL AORTIC ANEURYSM (AAA) REPAIR OR ANY OPERATION OF ABDOMINAL AORTA THEN WHAT COMPLICATIONS ARE SPECIFIC TO THEMe

BOWEL ISCHEMIA ********* - results from INADEQUATE COLONIC COLLATERAL ARTERIAL PERFUSION TO LEFT + SIGMOID COLON AFTER LOSS IMA DURING AORTIC GRAFT PLACEMENT - patients present with ABDOMINAL PAIN + BLOODY DIARRHEA - FEVER + LEUKOCYTOSIS - "TENDERNESS LLQ WITHOUT REBOUDN - USUALLY INDICATES NO PERITONEAL SIGNS SO NO RUPTURE"

Truck driver comes to ED. He had serious burns due to truck flipping + catching on fire. No PMH, no meds. BP of 110/70 + pulse of 98/min. PE reveals second degree burns covering both UE + 3rd deg burns on LE. On day 3 of hospitalization - develops tachy + dec UO. BP is 90/60 + pulse of 120/min, temp is 95F, RR of 26. Lab analysis reveals: glucose of 230, WBC of 16,000, platelet count of 80,000. Explanation?

Bacterial Infection

Hemodynamic Measurement in Shock

Cardiogenic Shock - issue is with the pump (heart) - Cardiac Index is dec - SVR + HR INC to compensate - dec mixed venous/O2 sat b/c less pumping going on - PWCP (LAP aka preload) + RAP (preload) - INCREASED b/c blood flow is constant + no loss of fluids - just heart pump isn't functioning Hypovolemic Shock - there is decreased volume due to loss from a port somewhere - therefore, PRELOAD DEC (PCWP + RAP DEC) - Cardiac Index is DEC - b/c less fluid to pump - SVR - increases to compensate - mixed venous oxygen saturation is decreased b/c less pumping Septic Shock - there is EXTRAVASATION DUE TO INFLAMM MEDIATORS - LOSS OF FLUIDS PERIPHERALLY + AT END ORGANS - DEC SVR - heart trying to perfuse end organs SO INC CARDIAC INDEX - INC PUMP FXN - Mixed O2/Venous Sat INC DUE TO INC PUMP FXN - PRELOAD DEC due to loss of fluid fundamentally to 3rd spacing

Ulnar Nerve Injury

Cause - fracture MEDIAL EPICONDYLE OF HUMERUS OR DISTAL LACERATION ANTERIOR WRIST Symptoms - associated with humeral fracture - claw hand - from paralysis of intrinsic muscles of hand - sensory loss of dorsal + ventral medial hand f

Pulmonary Complications of Postop

Complications - atelectasis (POST OP DAY 2 -5****), infection i.e. pneumonia - bronchospasm, exacerbation of chronic lung disease - prolonged mechanical ventilation Cause of Postoperative Atelectasis**** (TEST Q) - shallow breathing + weak cough due to pain - this is why pain control w/opioids is so important postop RFs - age > 50 years - Emergency surgery or surgery duration > 3 hours - Heart Failure, COPD - poor general health (ASA >2) Preoperative Strategies to reduce Risk - Smoking Cessation AT LEAST 8 WEEKS PRIOR TO SURGERY - SX CONTROL OF COPD - preop glucocorticoids if not well controlled - patient education for lung expansion maneuver - chest physical therapy***, DEEP BREATHING EXERCISES***, INCENTIVE SPIROMETRY*** Postoperative Strategies - INCENTIVE SPIROMETRY*** + DEEP BREATHING EXERCISES*** - BEST - epidural analgesia instead of parenteral opiods - continuous positive airway pressure (CPAP) - ADEQUATE PAIN CONTROL W/OPIOIDS - EARLY MOBILIZATION - DIRECTED COUGHING **** KNOW THE ONES IN CAPS - TEST Q***

Respiratory Quotient

Definition - ratio of CO2 produced to RATIO OF OXYGEN UPTAKE - - shows METABOLISM - DEPENDENT ON FUEL TYPE USED (LIPID VS CARBS VS ETC) - FA ALONE - RATIO OF 0.7 - PROTEINS ALONE - RATIO OF 0.8 - CARBS MAJOR NUTRIENT - RATIO of 1.0 - NORMAL RQ - 0.8 - In SEPSIS - RATION < 1 B/C FAT + PROTEIN ARE MAJOR SOURCES OF ENERGY

Diaphgragmatic Rupture

Diaphgramatic Rupture i. look for acute hx of BLUNT TRAUMA ii. Blunt abdominal trauma -can cause sudden increase in intraabdominal pressure that overcomes muscular strength of diaphgram + leads to large radial tears in muscle iii. diaphragmatic rupture - SHOULD SEE ABD CONTENT INTO CHEST iv. MORE COMMON ON LEFT SIDE - RIGHT SIDE PROTECTED BY LIVER v. presentation - suspect in patients with HX OF BLUNT TRAUMA/MOTOR vehicle accident, ABNROMAL CHEST X RAY LEFT LOWER LUNG OPACITY, ELEVATED HEMIDIAPHRAGM, HERNIATION OF ABDOMINAL ORGANS vii. Another presentation - CHILDREN MAY NOT PRESENT INITIALLY W/SX - DELAYED DX IS ASSOCIATED W/HIGH MORBIDITY - DUE TO RISK OF STRANGULATION *** vii. CXR - elevation of HEMIDIAPHRAGM ON CXR - MAY OR MAY NOT SEE BOWEL GO THRU THORACIC CAVITY ON LEFT SIDE; MAY ALSO SEE NG TUBE TRACE BACK IN TO PULMONARY CAVITY INDICATING Diaphragmati HERNIA; DEVIATION OF MEDIASTINAL CONTENTS TO OPPOSITE SIDE - Diagnosis - CXR SHOWING NG TUBE IN PULMONARY CAVITY; DO CT OF CHEST + ABDOMEN TO CONFIRM AFTER CXR (NEXT STEP AFTER CXR IF ASKED) (TEST Q****)

72 year old man undergoes CABG for severe CAD. Extubated 2nd postop day + temp is 97.9. BP is 127/80 mm Hg, pulse of 80/min, RR of 12/min. 3rd postop day complains of dyspnea + worsening retrosternal pain w/ temp 101.5 + BP of 112/52 mm Hg, pulse of 125/min + irregularly irregular, and respiration are 28/min. Examination shows normal heart sounds. A small amount of cloudy fluid is present in sternal wound drain. His ECG shows AFIB w/RVR. An xray of Chest shows widening of mediastinum. Echo reveals a small amount of pericardial fluid. Laboratory studies show: Hb: 8.9 WBC: 16,300 Platelet Count: 512,000 Creatinine: 1.7 CPK: 430 Dx?

Dx - Acute mediastinitis - when mediastinum is widened - think 2 things i. ruptured aorta ii. mediastinitis -

50 y/o man comes to ED w/2 day hx of abd. pain. Pain is shifting, periodic, coming in waves. Vomited x3 over last several hours. Has not passed gas or bowel movement for last 3 days. On PE abdomen is distended to palpation in all 4 quadrants. No guarding or rebound. Bowel sounds are present. There is inc. intensity at peak of patient's wave of pain. Temp is 101.3F; BP is 110/60; RR is 24. Labs are as follows: Hb - 14.0; WBC - 14,300; BUN - 36; pH = 7.26 AXR show multiple air fluid levels - NG tube is placed, IV access obtained, fluid resuscitation is done. What is next?

Dx - Small Bowel Obstruction Urgent Surgical Exploration

45 y/o woman undergoes elective total hysterectomy for uterine fibroids. In the postoperative recovery room, she develops nausea, vomiting, and acute abdominal pain. The patient has a history of SLE, pernicious anemia, TIDM, lower back pain, uterine fibroids. Her otpatien meds included vit B12, insulin, prednisone, hyrdoxychloroquine, and acetaminophen. Her blood pressure is 70/40 mm Hg and pulse is 110/min. BP improves to 90/60 after 4L of IV fluids. Pt has round plethoric face, buffalo hump, and central obesity. Lab results: - Na+ of 132, K+ of 4.9, Glucose of 50

Dx - acute adrenal crisis - most likely has central adrenal insufficiency due to chronic prednisone use for SLE. - Daily prednisone > 20 mg taken for > 3 weeks can suppress HPA OR patients with Cushingoid features i.e. buffalo humps, a. these patients above DO NOT RESPOND WELL TO STRESS - require HIGHER DOSES OF SHORT TERM GLUCOCORTICOIDS DURING ACUTE CONDITION IE SURGERY OR INFXN - Pts taking < 5 mg of prednisone have NO RISK OF HPA SUPPRESSION - DO NOT REQURIE stress dose steroids. - Hints of adrenal suppression i. hypoglycemia, hyponatremia, hyperkalemia, hypotension, ABNORMAL N/V/unexplained Abdominal Pain

How to manage HEMODYNAMICALLY USNTABLE PATIENT WITH PENETRATING ABDOM TRAUMA?

EMERGEN EXPLORATORY LAPAROTOMY - to dx + tx source of bleeding

Any penetrating wound below 4th intercostal space (level of nipples) is considered to involve the abdomen and requires what next step?

EMERGENT EXPLORATORY LAPAROTOMY - IF UNSTABLE

Prosthetic Joint Infection

Early Onset Infection - timing: within 3 months of primary arthroplasty - presentation: wound drainage, erythema, swelling often with fever - most common organisms: staphylococcus aureus, gram negative rods, anerobes - management: implant removal/exchange, may consider debridement + implant retention Delayed Onset of Infection - timing: >3 months after primary arthroplasty - presentation: persistent join pain, implant loosening or sinus tract formation - most common organisms: coagulase-negative staphylococci (STAPH EPIDERMIDIS), propionobacterium species - management: implant removal/exchange generally recommended *** STAPH EPIDERMIDIS IS MOST COMMON CAUSE OF DELAYED ONSET OF PROSTHETIC INFECTION

29 y/o W brought to ED after burning her RUE in cooking accident. Exam shows circumferential burn of RUE. Given fluids + dressing. ON day 3 she develops severe deep tissue pain in right limb w/edema of hand. Exam shows circumferential eschar over right arm. Right Radial + ulnar pulses are faint compared to left + she has paresthesias in her right hand. Which of following is most appropriate next step?

Escharotomy***

Management of gallstones

Gallstones without Sx - No Tx necessary in most patients Gallstones with typical biliary colic sx i. elective laparoscopic cholecystectomy ii. Possible URSODEOXYCHOLIC ACID IN POOR SURGICAL CANDIDATES Complicated Gallstone Disease: Acute Cholecystitis, choledocholithiasis, gallstone pancreatitis - Cholecystectomy w/in 72 hours

Common Peroneal Nerve (Fibular Nerve)

Gives rise to superficial peroneal + deep peroneal nerves

31 y/oM brought to Emergency room after motor vehicle collision - given 3 L of normal saline in ambulance on his way to hospital - been receiving 5L/min of O2 by nasal cannula. Is agitated + moves all 4 extremities spontaneously. BP is 85/55 mm Hg + HR is 120/min. RR is 30/min. Pupils are symmetric + reactive to light. Neck veins flat + trachea is shifted slightly to right. Over left hemithorax breath sounds are absent + there is dullness to percussion percussion. What is diagnosis?

Hemothorax Reasoning - presentation is hypotensive - NECK VEINS ARE FLAT - VOLUME DEPLETION - Patient is hypotensive even after receiving 3L of IV crystalloid - again tells us FLUID BEING LOST SOMEWHERE - NO SIGN OF EXT HEMORRHAGE SO HEMORRHAGE IS INTERNAL - mediastinal shift - tracheal shift tell us there is probably fluid in the region of thorax - THINK HEMOTHORAX OR PLEURAL EFFUSION - ALSO SUPPORTS FINDINGS OF DEC BREATH SOUNDS + DULLNESS TO PERCUSSION OVER INVOLVED SIDE - DIFF DX of PNEUMOTHORAX - yes hypotension, tachycardia, tachypnea, w/tracheal deviation opposite of defect; dec breath sounds on affected side BUT HYPERRESONANCE TO PERCUSSION OVER LEFT HEMITHORAX - AFTER BLUNT CHEST TRAUMA - HEMORRHAGIC SHOCK ASSOCIATED W/DEC BREATH SOUNDS + DULLNESS TO PERCUSSION OVER ONE HEMITHORAX + CONTRALATERAL TRACHEAL DEVIATION - MOST LIKELY DUE TO LARGE IPSLATERAL HEMOTHORAX - Next step - CHEST TUBE INSERTINON -

Hemorrhagic Shock

Hypotension, tachycardia, flat neck veins, confusion, and cold extremities despite IV fluid resuscitation in a trauma patient indicates hypovolemic/hemorrhagic shock

What is the common antimicrobial prophylaxis given before surgery to prevent would infxs w/in 60 mins of procedure?

IV cefazolin

Obturator Nerve

Innervation - medial compartment of thigh (gracilis adductor longus, adductor brevis,, anterior portion of adductor magnus) - ALLOW ADDUCTION OF THIGH - Sensation to MEDIAL THIGH

Femoral Nerve Injury

Innervation - muscles of anterior compartment - quadriceps femoris, Sartorius, pectineus - responsible for KNEE EXTENSION + HIP FLEXION (LEG STRAIGHT RAISE) - sensation to ANTERIOR THIGH + MEDIAL LEG (via saphenous branch) (IMAGE FOR DISTRIBUTION)

Tibial Nerve Injury

Innervation - supplies muscles of posterior compartment of thigh, posterior compartment of leg, and plantar muscles of foot i. allows flexion of knee + plantar flexion of foot - tibial nerve provides sensation to leg (EXCEPT MEDIAL SIDE) + Plantar Foot

Tetanus Prophylaxis

Know image - "dirty wound" = contaminated with dirt, feces, or saliva OR severe (i.e. puncture wounds, avulsions, wounds due to crushing, burns, or frostbite)

Infection of Submandibular space is called?

Ludwig's angina - begin on floor of mouth + extend through submandibular + sublingual space into the tissues surrounding the airway. It does not commonly extend into the mediastinum. - note sublingual space is a division of the submandibular space

Supracondylar Fracture of Humerus

Mechanism - fall on an outstretched hand most common Complications - brachial artery injury *** - LOSS OF PULSES IN RADIAL A. + BRACHIAL A. PULSES - median nerve injury *** *** BRACHIAL A. INJURY + MEDIAN NERVE INJURY - MOST COMMON INJURIES BY THIS FRACTURE TYPE - cubitus varus deformity - compartment syndrome/Volkmann ischemic contracture (uncommon) i. infarction from compartment syndrome can lead to Volkmann contracture - only occurs when supracondylar fracture of humerus has associated FOREARM Fracture PE - FIRST STEP - ASSESS FOR DISTAL PERFUSION AT DISTAL BRACHIAL A + RADIAL A + MOTOR + SENSORY FXN IN MEDIAN NERVE (SENSORY OF LAT 3.5 FINGERS) BEFORE YOU CONSIDIER REDUCTION Tx - ANALGESIA + IMMOBIZATION (IF NEUROVASCULAR INJURY, ALIGNMENT + IMMOBILIZATION RESOLVES ISSUE)

Injury of Axillary Nerve

Mechanisms 1. anterior dislocation of humeral head from glenoid fossa during FORCEFUL EXTERNAL ROTATION + ABDUCTION OF ARM (i.e. basketball player blocking shot) 2. physical exam show prominence of acromion w/abn subacromial space where humeral head normally resides 3. Fullness of anterior shoulder on palpation Presentation i. paralysis of deltoid + teres minor muscles - ii. loss of sensation over upper lateral arm

When are prophylactic antibiotics needed prior to surgery?

ONLY IF TEHRE IS RESPIRATORY INFXN PREEXISTING

33 y/o M falls of bicycle handlebars while riding in park + comes to ED w/BP of 128/80, pulse of 80/min. PE - upper abdominal bruises. Abdomen is nondistended, soft, and mildly tender in epigastrium. Complete blood count is normal. Abdominal CT scan reveals no abnormalities. Pt is sent home with analgesic medications. He returns a week later with fever, shaking chills, poor appetite, and deep abdominal pain. Which of the following is most likely related to patient's sx?

Pancreatic Laceration

23 y/o M to ED post motor vehicle accident. Unresponsive at scene + intubated. Received 2 L of normal saline over last 20 mins. BP is 80/40 + responds to strong vocal stimuli + tactile stimuli via opening his eyes. Pupils RRR. Neck veins are distended. There are multiple bruises involving anterior chest + upper abdomen. CXR shows small left sided pleural effusion + normal cardiac contours. What is diagnosis?

Pericardial Tamponade

Pneumothorax Treatment?

Positive Airway pressure WITH CHEST TUBE (w/o chest tube, no improvement) ta

Urethral Injury

Posterior Urethral Injury - signs of POSTERIOR urethral injury i. blood at meatus ii. inability to void iii. high riding prostate Anatomy of Posterior Urethra - includes i. prostatic urethra ii. membranous urethra - when you see additionally a PERINEAL OR SCROTAL HEMATOMA, THINK CAUSE = PELVIC FRACTURE CAUSED BOTH URETHRAL INJURY + HEMATOMA - on suspicion of POSTERIOR urethral injury - DO RETROGRADE URETHROGRAM BEFORE INSERTION FOLEY CATHETER (blind insertion can cause infection of periurethral hematoma) - Tx of Posterior Urethral injury - after retrograde urethrogram - do URINARY DIVERSION VIA SUPRAPUBIC CATHETER WHILE PRIMARY INJURY + ASSOC HEMATOMAS ARE ALLOWED TO HEAL; after healing complete, repair residual damage Anterior Urethral Injury - in case of ANTERIOR urethral injury - IMMEDIATE SURGICAL REPAIR - urethra portion that is distal to urogenital diaphragm - causes may be: i. BLUNT TRAUMA TO PERINEUM AKA STRADDLE INJURIES ii. INSTRUMENTATION TO URETHRA - Findings/Presentation i. perineal tenderness or hematoma ii. normal prostate (vs "high riding prostate) iii. bleeding from urethra (versus blood at meatus) iv. retain ability to void (versus posterior urethral injury) v. may see delayed presentation - patient may first present when there is SEPSIS DUE TO EXTRAVASATION OF URINE INTO SCROTUM, PERINEUM, OR ABD WALL

65 y/o Caucasian male goes thru surgery CA bypass grafting but requires 10 units Packed RBCS during surgery. ON 2nd postop day he has JAUNDICE + afebrile + PHSYCIAL EXAM IS UNREMARKABLE. Only thing elevated was alk phos at 300. What is dx?

Postoperative Cholestasis

Venous Insufficiency

Presentation - bilateral lower extremity edema - stasis dermatitis Etiology - due to VENOUS VALVULAR INCOMPETENCE - venous stasis + pooling of venous blood - gives inc pressure in postcapillary venules - causing damage of capillaries giving loss of fluid, plasma proteins, and erythrocytes into tissue i. erythrocyte extravasation - gives CLASSIC HEMOSIDERIN DEPOSITION INDUCED COLORATION OF STASIS DERMATITIS - predisposition for MEDIAL LEG BELOW KNEE + ABOVE MEDIAL MALLEOLUS - early dz = xerosis; late dz = lipdermatosclerosis + ulcerations ii. pooling gives LOWER EXTREM EDEMA iii. inflammation of venules + capillaries - gives MICROVASCULAR DIZ + ULCERATIONS SUMMARY STATEMENT: edema, stasis dermatitis, and venous ulcerations results rom lower extremity venous insufficiency due to valve incompetence. Such disease classically occurs on the medial leg superior to the medial malleolus

Intraductal Papilloma

Presentation - classic presentation = bloody discharge from one nipple - generally no mass - as mass is small, soft, directly below nipple Etiology - benign breast disease - common in PERIMENOPAUSAL WOMEN 35-40 Dx - clinically - US no good because can't detect such small masses

Pancreatic contusion, crush injury, laceration, or transection

Presentation - fevers, chills, DEEP abdominal pain - GENERALLY SUGGEST ABDOMINAL ABSCESS - any form of blunt abdominal trauma can compress neck/body of pancreas against vertebral column over which it lies. - pancreatic contusion, crush injury, laceration, or transection may result from BAT (blunt abd. trauma) - Abd CT may fail to detect - MUST DO SERIAL CTs TO DETECT - serum amylase fluids are non specific - untreated pancreatic injury not seen on single CT if missed may be complicated by RETROPERITONEAL ABSCESS OR PSEUDOCYST - treatment of retroperitoneal abscess - percutaneous drainage via catheter + surgical debridement

Bladder Injury

Presentation - gross hematuria - difficulty with urination - NO FINDING OF DISTENDED BLADDER - Intraperitoneal Bladder rupture - cause peritoneal signs (IE AT BLADDER DOME - most likely to rupture - will present with referred pain to shoulder due to rupture at this part causing pee to push up on hemidiaphragm)

Retroperitoneal Hematoma

Presentation - often present AFTER RECENT CARDIAC CATHETERIZATION - look for recent anticoagulation with heparin - HYPOTENSION + FLAT NECK VEINS = SUDDEN HEMODYNAMIC INSTABILITY *** - IPSLATERAL FLANK OR BACK PAIN*** ***** THE TWO SX W/*** NEXT TO THEM ARE DIAGNOSTIC CLINICALLY WHEN SEEN TOGETHER Etiology - due to bleeding form ARTERIAL ACCESS SITE (WITH RETROPERITONEAL EXTENSION) - when arterial ACCESS SITE IS ABOVE INGUINAL LIGAMENT, HEMATOMA MAY SPREAD RETROPERITONEALLY Dx - NON CONTRAST CT ABDOMEN + PELVIS Tx - supportive; IV fluids or blood transfusion Preventative Care - catheterized pts for fear of formation of hematoma are advised TO NOT LIFT HEAVY OBJECTS FOR 1 WEEK POST CATHETERIZATION - RADIAL A - LEAST VASCULAR COMPLICATIONS

Anal Fissure

Presentation - slit like tears of anal canal - located in posterior or anterior anal verge - BRB during defecation - severe pain Etiology - passage of hard + large constipated stools - other causes - explosive diarrhea, perianal dermatitis, or infection, IBD, trauma, and sexual abuse Tx - STOOL SOFTENER + LOCAL ANESTHETIC + DIETARY MODIFICATION (high fiber diet + large amounts of fluids) - if chronic or refractory acute fissures - LATERAL SPHINCTEROTOMY = STD TX

Stress Fracture

Presentation 1. excess exercise 2. dull aching pain at bone, superficially 3. Point tenderness (i.e. over anterior aspect of middle shin) 4. PE unremarkable 5. XR - no abnormalities 6. Common in BALLET DANCERS, BASKETBALL PLAYERS, SOCCER PLAYERS, and MILITARY RECRUITS 7. Tibia Stress Fracture - MOST COMMON SITE i. common place for MEDIAL TIBIAL STRESS SYNDROME - "shin splints" w/no tibial tenderness to palpation ii. can progress w/more activity to complete/incomplete fracture (stress fracture) giving pain to palpation iii. assoc w/running or jumping iv. x ray - frequently normal; may reveal PERIOSTEAL RXN IN SITE OF FRACTURE v. injury best defined WITH MRI OR BONE SCAN *** vi. Tx = REST + HEAL BONE FRACTURE 8. Most commonly stress fractures will occur in MIDDLE THIRD OF TIBIA in pts involved in jumping sports + POSTEROMEDIAL PART OF DISTAL THIRD OF TIBIA IN RUNNERS

Acute Pericardial Tamponade

Presentation 1. hypotension unresponsive to IV fluid bolus 2. Tachycardia 3. Distended Neck Veins (elevated JVP) 4. all of these after BLUNT THORACIC TRAUMA 5. Beck's triad - if you see MUFFLED HEART SOUNDS, HYPOTENSION, DISTENDED NECK VEINS 6. Often see left sided pleural effusion (trauma induced) Cause 1. myocardial rupture Imaging 1. CXR - MAY APPEAR NORMAL 2. DX TEST OF CHOICE - TRANSESOPHAGEAL ELECTROCARDIOGRAM i. will reveal ELECTRICAL ALTERANS - varying amplitude of QRS complexes Resulting Pathophysiology 1. Cardiogenic Shock Treatment 1. IMMEDIATELY WITH DECOMPRESSION VIA PERICARDIOCENTESIS OR SURGICAL PERICARDIOTOMY

Alveolar Hypoventilation

Presentation - ABG w/dec pH < 7.35 - primary inc. in PaCO2 > 40 mm Hg - ****High PaCO2 (SPEC IF IN RANGE OF 50-80 mm Hg) + low PaO2 AFTER OPERATION - THINK ALVEOLAR HYPOVENTILATION Causes of Alveolar Hypoventilation - pulmonary/thoracic disease - COPD, OSA, Obesity hypoventilation, scoliosis - neuromuscular disease - myasthenia gravis, Lambert eaton syndrome, guillain barre syndrome - drug induced hypoventilation - anesthetics, narcotics, sedatives - primary CNS dysfunction - brainstem lesion, infection, stroke Aa Gradient - is within normal means - <15 - values inc. w/age but A-a gradient >30 is considered elevated - A-a = PAO2 - PaO2 = 76-PaO2 Causes of Elevated A-a gradient > 30 - differential diagnosis (b/c A-a in alveolar hypoventilation is normal (<14) i. PE ii. atelectasis iii. Pulmonary edema iv. pleural effusion - A-a > 30 - indicative that there is V/Q mismatch

Acute Diverticulitis

Presentation - LLQ pain - diverticula present on colonoscopy - fever + leukocytosis - CT scan shows inflammatory changes + soft tissue stranding + colonic wall thickening Classification - complicated - refers to association of diverticulitis with i. perforation a. management - sigmoid resection ii. obstruction a. management - sigmoid resection iii. fistula formation a. management - sigmoid resection iv. abscess a. presentation: 1. on CT fluid collection or "ring enhancing fluid collection" 2. LLQ tenderness w/o guarding or rebound - return after tx acute diverticulitis w/persistent abd pain, N/F b. management: 1. if <3cm - IV antibiotics, + observation 2. if >3cm - CT GUIDED PERCUTANEOUS DRAINAGE (TEST Q) 3. if sx not controlled by 5th day - surgical drainage + debridement - uncomplicated i. management: ORAL antibiotics, OBSERVATION, BOWEL REST

Retropharyngeal Abscess

Presentation 1. neck pain 2. fever of 101 or 102 3. limited neck mobility secondary to pain - DIFFICULTY ON EXTENSION 4. Trusmus - inability to open mouth normally 5. difficulty swallowing - dysphagia + pain on swallowing (odynophagia) Etiology 1. usual source of infection is from LOCAL PENETRATING TRAUMA i. due to instrumentation or following injury from some type of bone Evaluation 1. CT **** - should see lordosis cervical spine w/gas + swelling of retropharyngeal space Treatment 1. IV broad spectrum antibiotics + URGENT DRAINAGE ABSCESS (TO PREVENT SPREAD TO MEDIASTINUM AND ADVANCEMENT TO MEDIASTINITIS)

Acute Colonic Pseudoobstruction

Presentation 1. result from trauma + 2. present w/N + ABD pain, 3. ABD Distention, 4. Tympanitic Bowel Sounds + Hyperactive Bowel Sounds (LIKE SBO BUT IN LARGE BOWEL) 5. AXR - show massively dilated colon w/o sign. Small Bowel Dilation - due to patent ileocecal valve

Ischemic Colitis

RFs - age > 60 - chronic renal dz/hemodialysis - atherosclerotic vessel dz + procedures - MI CFs - mild pain + tenderness - hematochezia, diarrhea - metabolic (LACTIC) acidosis Dx - CT: thickened bowel wall****, double halo sign, pneumatosis coli Colonoscopy: Mucosal pallor or cyanosis, petechial, hemorrhage; CYANOTIC SEGMENTS OF MUCOSA*** + HEMORRHAGIC ULCERATION*** w/SHARP TRANSITION FROM AFFECTED TO UNAFFECTED MUCOSA Management - supportive care: IV fluids, bowel rest - IV antibiotics - IF PERFORATION OR GANGRENE - then Colon resection (bowel infarct or clinical deterioration) - classic presentation - abdominal pain + bloody diarrhea following vascular procedure w/pain lateralizing to affected side - often due to DEC FLOW THRU IMA***

Pancreatic Adenocarcinoma

RFs - smoking - hereditary pancreatitis - nonhereditary pancreatitis - obesity + lack of physical activity Clinical Presentation - systemic symptoms (weight loss, anorexia, fatigue) (>85%) - "GNAWING EPIGASTRIC PAIN WORSE AT NIGHT - abd pain/back pain (80%) - jaundice (56%) - STEATORRHEA - inability to sec fat digesting enzymes or blockage in main pancreatic duct - recent onset atypical diabetes mellitus - unexplained migratory superficial thrombophlebitis (TROSSEAU SIGN) - hepatomegaly + ascites w/mets Lab Studies - cholestatic picture - inc alk phos + direct bilirubin - due to common bile duct obstruction ***** - elevated CA 19-9 (NOT AS A SCREENING TEST) Dx Test - US (if jaundiced) or CT (IF NOT jaundiced) ******

45 y/o m to ed w/motor vehicle accident. Unable to void w/blood at urethral meatus + scrotal hematoma. Temp of 98.6 + r of 16/min. Exam - high riding prostate w/no signs of trauma. What is next step?

Retrograde urethrogram

22 y/p football player comes w/difficulty extending right knee. Started 1 month ago when he twisted knee + now knee motion is limited. PE shows no swelling of knee but difficuluty in extending it fully. While passively flexed + extended, popping sensation is noted under the examiner's fingers (placed at medial + lateral join lines of right knee). Which of the following is the most appropriate next step in management?

Right Knee MRI

Common Causes of Shoulder Pain

Rotator Cuff Impingement or Tendinopathy - pain with ABDUCTION, EXT ROTATION - subacromial tenderness - normal range of motion with (+) impingement tests (eg, NEER, Hawkins) i. NEER test - internal rotate at humerus, stabilize pt scapula with one hand, passively raise arm through forward flexion @ humerus to see for pain - this indicates impingement between greater tuberosity of humerus + inferior acromion ii. Hawkins test - elbow + shoulder are flexed passively and at this point, arm rotated at humerus - same meaning as NEER test - repetitive activity above shoulder height (e.g., painting ceilings) - supraspinatus is most susceptible *** - impingement syndrome - characteristic of RCT - refers to compression of either supraspinatus tendon or subacromial bursa Rotator Cuff Tear - similar to rotator cuff Tendinopathy - weakness with external rotation - age > 40 Adhesive Capsulitis (frozen shoulder) - decreased PASSIVE + ACTIVE ROM - more stiffness than pain Biceps Tendinopathy/rupture - ANTERIOR shoulder pain - pain with lifting, carrying, overhead reaching - weakness is LESS COMMON Glenohumeral osteoarthritis - uncommon + usually caused by trauma - gradual onset of anterior or deep shoulder pain - dec active + passive abduction + external rotation

Small Bowel Obstruction versus Ileus

SBO - Etiology - prior surgery (ABD) (weeks to years) - Abd Examination - (distention, INC BOWEL SOUNDS) - Small Bowel Dilation: PRESENT - Large Bowel Dilation: ABSENT - Obstipation - NO PASSAGE GAS AND NO PASSAGE STOOL ***ADHESIONS = MOST COMMON CAUSE FROM ABD SURGERY - typical presentation i. colicky abd pain, vomiting, no bowel movement or passing of gas (obstipation), abd distention, diffuse tenderness ii. N/V more common in SBO than Large bowel obstruction - B/C ILEIOCECAL VALCE IS COMPETENT iii. contents - more bilious w/proximal SBO vs more feculent in distal SBO iv. INC BOWEL PAIN W/INC BOWEL SOUNDS during PERISTALTIC RUSH Classifications - proximal vs distal i. proximal - early vomiting, abd discomfort, abn contrast filling on xray ii. distal/mid - obstipation, hyperactive bowel sounds, distal loops of small bowel on abd x ray - strangulated vs simple i. simple - luminal occlusion ii. strangulated - peritoneal signs of guarding, rigidity, rebound + signs of shock (LATE - fever, tachy, leukocytosis) - Labs i. any bowel obstruction - MODEST INC AMYLASE ii. MILD LEUKOCYTOSIS - Management i. Bowel Rest, Decompression w/NG, IV fluids, correction of metabolic derangements ii. HOWEVER: IF PRESENTING WITH FEVER, TACHY, LEUKOCYTOSIS (SIG), AND/OR METABOLIC ACIDOSIS THEN - EMERGENT LAPAOSCOPY (EXPLORATION) (EXAMPLE IN LATER NOTES) Ileus - Etiology - RECENT (ABD) surgery (HOURS TO DAYS), METABOLIC (HYPOKALEMIA), or MED INDUCED, or PANCREATITIS INDUCED - also may occur after TRAUMA CONDITIONS - i.e. retroperitoneal hemorrhage after vertebral fracture - a disruption of normal neurologic + motor control of GI tract - Abd Examination - Possible DISTENTION, REDUCED/ABSENT BOWEL SOUNDS (VS) - Small Bowel Dilation: PRESENT - Large Bowel Dilation: PRESENT (VS) - NO PASSAGE OF EITHER GAS OR STOOL - NOT BOTH (VS); abd distention - Management i. conservative - bowel rest, supportive care, and treatment of any secondary causes of ileus - MORPHINE + OPIATES COMPOUND THIS PROBLEM BY DEC GI MOTILITY (TEST Q***)

What is SIRS vs Sepsis?

Sepsis 1. clinical syndrome characterized by systemic inflammation + tissue injury 2. usually an original insult - IN SEPSIS, INSULT MUST BE INFECTIOUS - giving inflammation + dysregulated host response - mass rel. proinflamm. substances causing extensive tissue dmg. 3. "SIRS" with known infectious cause 4. "Serious sepsis" - when there is associated END ORGAN DYSFXN i. thrombocytopenia (<80,000) ii. hypotension w/systolic < 90 iii. Metabolic acidosis iv. hypoxemia SIRS - clinical syndrome characterized by systemic inflammation + tissue injury - usually an original insult - IN SIRS, INSULT MUST BE NON INFECTIOUS - giving inflammation + dysregulated host response - mass rel. proinflamm. substances causing extensive tissue dmg. i. pancreatitis ii. AI dz iii. vasculitis iv. burns - criteria: MUST HAVE AT LEAST 2 OF 4 i. Temp > 101.3 (38.5) or <35 (95) ii. pulse > 90/min iii. RR > 20/min iv. WBC > 12,000 OR <4000 OR >10% bands

69 y/o male evaluated hours after undergoing elective repair of descending thoracic aortic aneurysm. Has weakness in both LE + urinary retention, which were not present immediately after procedure. Surgery was complicated by significant blood loss + req multiple transfusions. Pt has a history HTN, hyperlipidemia, TIIDM, CAD. 70% stenosis of RCA. Neruo exam shows flaccid paraplegia + loss of pain sensation over the LE. Vibratory sensation is intact. UE exam shows no abnormalities. Which of the following is cause of his neurologic exam?

Spinal cord infarction - resulting in anterior cord syndrome

What is most feared complication of Retropharyngeal abscess?

Spread of infection into mediastinum - giving acute necrotizing mediastinitis - tx = early dx + debridement of mediastinum to treat this complication

which type of cancer is associated with a burn that leads to a chronic draining wound that never closes from the burn?

Squamos cell Cancer - most often associated with UV exposure - may arise from chronically wounded, scarred, inflamed skin. SCC arising within a burn wound is known as Marjolin Ulcer. - additionally, SCC - seen w/skin OVERLYING FOCUS OF OSTEOMYELITIS, RADIOTHERAPY SCARS, AND VENOUS ULCERS - if it arises in chronic wound, more aggressive - even more important to EARLY DX W/BIOPSY

Stages of Hemorrhagic Shock

Stage I - compensation - occurs via sympathetic response inducing MILD TACHYCARDIA + PERIPHERAL VASOCONSTRICTION (INC SVR) (KNOW RESPONSE LISTED) Stage II - compensation - pulse rate even more elevated, BP begins to trend downward, UO starts to fall - skin is cool and moist (not mentioned in table) + capillary refill delayed - duet to shunting of blood from less critical vascular beds such as skin, leading to skin vasoconstriction When Hemorrhage occurs -TACHYCARDIA + PERIPHERAL VASOCONSTRICTION (SVR) ARE 1st PHYSIOLOGIC RESPONSES - ACT TO MAINTAIN BP

Dumping Syndrome

Sx - abdominal cramps, diarrhea, nausea + vasomotor (PALPITATIONS + DIAPHORESIS) Etiology - POST GASTRECTOMY COMPLICATION - due to LOSS OF NORMAL ACTION OF PYLORIC SPHINCTER - due to INJURY OR SURGICAL BYPASS - gives early emptying of hypertonic gastric contents into duodenum + small intestine i. causes resulting fluid shift from intravascular space to SMALL INTESTINE - gives HYPOTENSION, RESULTING IN INC ANS GIVING PALPITATIONS + DIAPHORESIS (VASOMOTOR UPREGULATION + RELEASE OF VASOACTIVE POLYPEPTIDES) Dx - CLINICAL, NO IMAGING NEEDED Timing - 15-30 mins after meals Pathogenesis - rapid emptying of hypertonic gastric contents Initial Management - ALL DIETARY MANAGEMENT i. smaller, frequent meals ii. replace simple sugars w/complex carbs iii. incorporate high fiber + protein rich foods

Meniscal Injury

Sx - pain worse on twisting movement - popping sensation - popping, catching knee giving out, and locking (inability to extend knee) Cause - TWISTING INJURY OF FOOT in FIXED POSITION PE Findings - joint line tenderness - loss of smooth flexion or extension - inability to move forward or backward while squatting - effusion - McMurray test - painful click on passive flexion + extension of knee w/examiner's thumb + index fingers placed at medial + lateral joint lines. - Apley Test - pain w/pressing heel toward ground while internally + externally rotating foot w/knee flexed to 90 degs. Preferred Test - MRI***** - surgery (arthroscopic or open) needed to fix

62 y/o man presents to ED w/1 hours of severe epigastric pain. He has been having some epigastric pain exacerbated by eating for last several days. He feels nauseated + has vomited once since pain began. PMH sig for HTN, diabetes, hyperlipidemia, CAD. Went through CABG one year ago. Current meds are simvastatin, aspirin, clopidogrel, Metoprolol, enalapril. BP of 140/100 + HR of 65, ecg is normal. CXR shows under diaphragm. What is dx? Next Step?

Symptoms with eating worse seems to make it gastric ulcer + air under diaphragm tells us it has perforated. Next step - SURGICAL EXPLORATION

5 y/o M c/o muscle weakness + sensory loss in UE. PMH w/significant involvement in a motor vehicle accident 7 years again which he sustained whiplash cervical spine injury. PE today reveals moderate wasting of small hand muscles + impaired pain + Temperature sensation in the bilateral UE. Light touch, vibration, and position senses are all intact. What is dx?

Syringomyelia

Dx test of choice for hemopericardium?

TEE

Follicular Occlusion Tetrad Dz

Tetrad 1. pilonidal dz 2. suppurative hidradenitis 3. dissecting folliculitis of the scalp 4. acne conglobata Presentation - painful nodules + pustules of axillae + groin - lesions lead to sinus formation + fibrosis

Hypotension + Distended Neck Veins Differential

Think Either 1. Pericardial Tamponade 2. Tension Pneumothorax

Aortoiliac Occlusion (Leriche Syndrome)

Triad 1. bilateral hip, thigh, buttock claudication 2. impotence 3. symmetric atrophy of bilateral lower extremities due to chronic ischemia - MUST HAVE IMPOTENCE - OTHERWISE, DX DOES NOT FIT RFs - men w/predisposition for atherosclerosis - smokers **** MOST IMPT

In middle age adult - superficial unilateral hip pain that is exacerbated by external pressure to the upper lateral thigh (as when lying on the affected side in bed) suggests what?

Trochanteric bursitis

Pediatric Abdominal Wall Defects

Umbilical Hernia - due to incomplete closure of abd muscles around umbilical ring at birth - most commonly associated w/ i. AA race ii. Ehler danlos syndrome iii. Premature Birth iv. Beckwith WIedemann Syndrome v. Hypothyroidism - PE i. soft, non tender bulge covered by skin that protrudes during crying, coughing, or straining ii. reduced easily through the umbilical ring w/very low risk of incarceration + strangulation - Tx i. small umbilical hernias - close spontaneously by concentric fibrosis + scar tissue formation ii. if large (>1.5 cm diameter) - less likely to close iii. if at age > 5, persistent hernia, then SURGICAL CLOSURE

Burn Victims Intubation Indications

Uworld Summary Statement - in urn victims, clinical indicators of thermal inhalation injury to upper airway +/or smoke inhalation injury to lungs includes burns to the face, singing of the eyebrows, oropharyngeal inflammation/blistering, oropharyngeal carbon deposits, carbonaceous sputum, stridor, carboxyhemoglobin level > 10%, or history of confinement in a burning building. Presence of >1 of these indicators warrants EARLY INTUBATION TO PREVENT UPPER AIRWAY OBSTRUCTION VIA EDEMA

Most common complications of Cardiac Catheterization

all are local vascular complications 1. bleeding 2. hematoma - localized or extended retroperitoneally 3. arterial dissection 4. AV fistula 5. Acute thrombosis 6. pseudo aneurysm *** HEMORRHAGE OR HEMATOMA FORM WITHIN 12 HOURS OF CATHETERIZATION


Related study sets

Accident and Health Insurance Basics

View Set

Chapter 13 REVEL: theories of personality

View Set

Chapter 34: Caring for Clients with Immune-Mediated Disorders

View Set

Chapter 18: Toward a New World View

View Set

ECON Lowdown Credit Bureaus: The Record Keepers

View Set